*NURSING > TEST BANK > Primary Care: Art and Science of Advanced Practice Nursing - An Interprofessional Approach 5th editi (All)

Primary Care: Art and Science of Advanced Practice Nursing - An Interprofessional Approach 5th edition Dunphy Test Bank Latest for Grade A+

Document Content and Description Below

Primary Care: Art and Science of Advanced Practice Nursing - An Interprofessional Approach 5th edition Dunphy Test Bank Chapter 1. Primary Care in the Twenty-First Century: A Circle of Caring 1. ... A nurse has conducted a literature review in an effort to identify the effect of handwashing on the incidence of nosocomial (hospital-acquired) infections in acute care settings. An article presented findings at a level of significance of <0.01. This indicates that A) the control group and the experimental group were more than 99% similar. B) the findings of the study have less than 1% chance of being attributable to chance. C) the effects of the intervention were nearly zero. D) the clinical significance of the findings was less than 1:100. Ans: B Feedback: The level of significance is the level at which the researcher believes that the study results most likely represent a nonchance event. A level of significance of <0.01 indicates that there is less than 1% probability that the result is due to chance. 2. A nurse has read a qualitative research study in order to understand the lived experience of parents who have a neonatal loss. Which of the following questions should the nurse prioritize when appraising the results of this study? A) How well did the authors capture the personal experiences of these parents? B) How well did the authors control for confounding variables that may have affected the findings? C) Did the authors use statistical measures that were appropriate to the phenomenon in question? D) Were the instruments that the researchers used statistically valid and reliable? Ans: A Feedback: Qualitative studies are judged on the basis of how well they capture and convey the subjective experiences of individuals. Statistical measures and variables are not dimensions of a qualitative methodology. 3. A nurse has expressed skepticism to a colleague about the value of nursing research, claiming that nursing research has little relevance to practice. How can the nurses colleague best defend the importance of nursing research? A) The existence of nursing research means that nurses are now able to access federal grant money, something that didnt use to be the case. B) Nursing research has allowed the development of masters and doctoral programs and has greatly increased the credibility of the profession. C) The growth of nursing research has caused nursing to be viewed as a true profession, rather than simply as a trade or a skill. D) The application of nursing research has the potential to improve nursing practice and patient outcomes. Ans: D Feedback: The greatest value of nursing research lies in the potential to improve practice and, ultimately, to improve patient outcomes. This supersedes the contributions of nursing research to education programs, grant funding, or the public view of the profession. 4. Tracy is a nurse with a baccalaureate degree who works in the labor and delivery unit of a busy urban hospital. She has noticed that many new mothers abandon breast-feeding their babies when they experience early challenges and wonders what could be done to encourage more women to continue breast-feeding. What role is Tracy most likely to play in a research project that tests an intervention aimed at promoting breast-feeding? A) Applying for grant funding for the research project B) Posing the clinical problem to one or more nursing researchers C) Planning the methodology of the research project D) Carrying out the intervention and submitting the results for publication Ans: B Feedback: A major role for staff nurses is to identify questions or problems for research. Grant applications, methodological planning, and publication submission are normally carried out by nurses who have advanced degrees in nursing. 5. A patient signed the informed consent form for a drug trial that was explained to patient by a research assistant. Later, the patient admitted to his nurse that he did not understand the research assistants explanation or his own role in the study. How should this patients nurse respond to this revelation? A) Explain the research process to the patient in greater detail. B) Describe the details of a randomized controlled trial for the patient. C) Inform the research assistant that the patients consent is likely invalid. D) Explain to the patient that his written consent is now legally binding. Ans: C Feedback: Just as the staff nurse is not responsible for medical consent, the staff nurse is not responsible for research consent. If patients who have agreed to participate exhibit ambivalence or uncertainty about participating, do not try to convince them to participate. Ask the person from the research team who is managing consents to speak with concerned patients about the study, even after a patient has signed the consent forms. Multiple Selection 6. A nurse leader is attempting to increase the awareness of evidence-based practice (EBP) among the nurses on a unit. A nurse who is implementing EBP integrates which of the following? (Select all that apply.) A) Interdisciplinary consensus B) Nursing tradition C) Research studies D) Patient preferences and values E) Clinical expertise Ans: C, D, E Feedback: Fineout-Overholt, Melnyk, Stillwell, and Williamson define EBP as a problem-solving approach to the delivery of healthcare that integrates the best evidence from studies and patient care data with clinician expertise and patient preferences and values. Multiple Choice 7. Mrs. Mayes is a 73-year-old woman who has a diabetic foot ulcer that has been extremely slow to heal and which now poses a threat of osteomyelitis. The wound care nurse who has been working with Mrs. Mayes applies evidence-based practice (EBP) whenever possible and has proposed the use of maggot therapy to debride necrotic tissue. Mrs. Mayes, however, finds the suggestion repugnant and adamantly opposes this treatment despite the sizable body of evidence supporting it. How should the nurse reconcile Mrs. Mayes views with the principles of EBP? A) The nurse should explain that reliable and valid research evidence overrides the patients opinion. B) The nurse should explain the evidence to the patient in greater detail. C) The nurse should integrate the patients preferences into the plan of care. D) The nurse should involve the patients family members in the decision-making process. Ans: C Feedback: Patient preferences should be integrated into EBP and considered alongside research evidence and the nurses clinical expertise; evidence does not trump the patients preferences. The family should be involved, but this is not an explicit dimension of EBP. Similarly, explaining the evidence in more detail is not a demonstration of EBP. 8. The administrators of a long-term care facility are considered the use of specialized, pressure- reducing mattresses in order to reduce the incidence of pressure ulcers among residents. They have sought input from the nurses on the unit, all of whom are aware of the need to implement the principles of evidence-based practice (EBP) in this decision. Which of the following evidence sources should the nurses prioritize? A) A qualitative study that explores the experience of living with a pressure ulcer B) A case study that describes the measures that nurses on a geriatric unit took to reduce pressure ulcers among patients C) Testimonials from experienced clinicians about the effectiveness of the mattress in question D) A randomized controlled trial that compared the pressure-reducing mattress with standard mattresses Ans: D Feedback: The most reliable evidence is considered RCTs. Qualitative studies, case studies, and expert opinion are low on the hierarchy of evidence. 9. Hospital administrators are applying the principles of evidence-based practice (EBP) in their attempt to ascertain the most efficient and effective way to communicate between nurses who are on different units, a project that will consider many types of evidence. Which of the following information sources should the administrators prioritize? A) A systematic review about communication in nursing contexts B) Nurses ideas about communication methods C) The results of a chart review D) The hospitals accreditation status Ans: A Feedback: Systematic reviews are assigned a high value in EBP. Reviews would be prioritized over nurses ideas or a chart review, though both are potential considerations. The hospitals accreditation status is not a relevant consideration. 10. A nurse has resolved to apply the evidence-based practice (EBP) process to the way that admission assessments are conducted and documented on a unit. How should the nurse begin the process of establishing EBP? A) Gather evidence showing the shortcomings of current practices B) Formulate a clear and concise question to be addressed C) Elicit support from the nurses who are most often responsible for admissions D) Search the literature for evidence that is potentially relevant to the practice need Ans: B Feedback: The first step in applying EBP is to ask a clear, focused question. This should precede a search of the literature or the recruitment of participants. An assessment of the shortcomings of the current system is not an explicit component of the EBP process. 11. Which of the following questions best exemplifies the PICOT format for asking evidence-based questions? A) What affect does parents alcohol use have on the alcohol use of their teenage children? B) Among postsurgical patients, what role does meditation rather than benzodiazepines have on anxiety levels during the 48 hours following surgery? C) Among high school students, what is the effectiveness of a sexual health campaign undertaken during the first 4 weeks of the fall semester as measured by incidence of new sexually transmitted infections? D) In children aged 68, is the effectiveness of a descriptive pain scale superior to a numeric rating scale in the emergency room context? Feedback: The correct answer includes a population (postsurgical patients), intervention (meditation), comparison (benzodiazepines), outcome (anxiety levels), and a time frame (48 hours). No other option contains each of the five elements of a PICOT question. 12. A nurse has made plans to implement the University of North Carolina (UNC) model of 5 As during the process of applying evidence-based practice (EBP) to a practice problem. What is the final step that the nurse will take in applying this model? A) Analyze the results of the EBP process B) Advocate for others to embrace the identified change C) Adopt the changes identified in the review process D) Assess the outcomes of the new practice Ans: D Feedback: The final step in the UNC rubric is to Assess the change using the quality improvement process in place in the institution. 13. A nurse has been asked to make a presentation to a group of high school students on the subject of sexual health. However, the nurse does not have a background in this practice area and requires rapid access to evidence-based guidelines. Which of the following strategies is most likely to provide the nurse with valid and reliable evidence in a time-efficient manner? A) Search the Cochrane Library of Systematic Reviews B) Google search terms such as sexual health teens and sexual education C) Search Medline using PubMed and order relevant articles D) Scan the most recent issues of nursing journals that address this area of practice Feedback: For some problems, a systematic review may be available from a source such as the Cochrane Library. Often this review is done by an expert panel providing excellent information on which to base decisions. This approach is more likely to produce valid and reliable results than a Google search and is more efficient than searching journal manually or ordering articles through PubMed. 14. The nurses at a university hospital have been informed that a computerized record system will be implemented over the next 12 months. The nurses should be aware that such as system presents particular challenges in the area of A) vulnerability to errors in charting and the inability to make changes. B) patient privacy and confidentiality of records. C) enforcing compliance with the system on the part of nurses. D) ensuring compatibility with different computer operating systems. Ans: B Feedback: Concerns about privacy become magnified when information is available to many people in many sites far removed from where the patient is located, a situation that exists when computerized records are used. This is usually considered a more important concern than issues of compliance, compatibility, or vulnerability to errors. 15. A nurse is nervous about the impeding introduction of computerized nursing care records at the hospital because he does not consider himself to be technologically adept. How should this nurse best respond to this situation? A) Take courses in advanced practice nursing to build his knowledge. B) Explore employment opportunities in settings that use written documentation systems. C) Advocate for a delay in the introduction of the proposed system. D) Seek out opportunities to learn the relevant knowledge and practice the necessary skills. Ans: D Feedback: A nurse who lacks technological knowledge or skills should seek out opportunities to expand these. This is preferable to finding a job elsewhere, studying advanced practice nursing, or attempting to delay the change. Chapter 2. Caring and the Advanced Practice Nurse Multiple Choice 1. A goal of community nursing is to provide primary prevention from disease. Which of the following nursing actions reflect this goal? A) A nurse creates a pamphlet discussing heart-healthy foods and distributes it in the neighborhood community center. B) A nurse starts an intravenous line on a dehydrated baby who has been brought to the emergency department. C) A nurse performs range-of-motion exercises for a patient in traction. D) A nurse repositions an elderly patient confined to a wheelchair to avoid the formation of pressure ulcers. Ans: A Feedback: Primary prevention involves the efforts to prevent disease from ever occurring. Primary prevention can be aimed at stopping the cause of disease. Generalized efforts to educate people regarding healthy diets are aimed at this type of primary prevention. Tertiary prevention focuses on preventing long-term disability and restoring functional capacity, as exemplified by repositioning an immobile patient, rehydrating a patient, or assisting with exercises. 2. A nurse decides to pursue a career in community-based nursing. Which of the following statements represents the environment in which the nurse will be working? A) Community-based nursing is limited to work in public clinics, schools, and industry. B) The key to community-based settings is that the nurse is in charge. C) The nurse serves as an educator, guide, and resource person and determines the action taken by the client. D) Care in the community is cost-effective. Ans: D Feedback: Care in the community is cost-effective and often more acceptable to the client because it causes less disruption in life. It takes place in a wide variety of settings and involves the nurse entering into a collaborative relationship with clients. 3. The movement of a client from acute care to a long-term nursing care facility involves planning to provide continuity of care. What is the term for this type of planning? A) Discharge planning B) Comprehensive planning C) Ongoing planning D) Transition planning Ans: D Feedback: Transitions are the movement of the patient from one care environment to another. Transition planning refers to the planning process that takes place to assure that the patients well- being is maintained throughout the time of transition. Organizing this transition from one care setting to another is not termed discharge planning, comprehensive planning, or ongoing planning. 4. A nurse is called into work to perform triage in the aftermath of an earthquake. Which of the following are the expected responsibilities of this nurse? A) Set up and monitor IV lines. B) Prepare the emergency room for multiple victims. C) Screen victims to prioritize treatment. D) Check available blood products and assist with transfusions. Ans: C Feedback: Triage involves the initial screening of victims for the purpose of prioritizing treatment and making the most effective and efficient use of both human and material resources. The other noted tasks are within the scope of disaster nursing but are not triage activities. 5. A client asks a nurse for help in obtaining an alternative healthcare provider. Which of the following is an accurate fact regarding alternative care that the nurse should share with this client? A) Most alternative healthcare practitioners do not have education-based credentials to practice their medicine. B) Alternative providers are not usually included in the federal HIPAA legislation that mandates confidentiality in conventional healthcare settings. C) The cost of alternative therapy is never covered by insurance carriers or healthcare plans. D) It is easy to find accurate safety and efficacy data for alternative medicine on the Internet. Ans: B Feedback: Alternative providers are not normally included in the federal Health Insurance Portability and Accountability Act of 1996 legislation that mandates confidentiality in conventional healthcare settings. Alternative practitioners do not necessarily lack credentials. Accurate online information can be difficult to find and costs for treatment may be covered by some insurance plans. 6. There is an increasing trend for nursing care to move from the hospital setting into the community. Nurses who are to provide excellent care in a community setting should prioritize which of the following? A) Integrating culture and family into the planning and delivery of care B) Becoming more assertive in client education and the planning of client care C) Encouraging clients to limit their interactions with physicians D) Teaching clients to replace biomedical interventions with complementary therapies Ans: A Feedback: The move to community care heightens the importance of family-centered, culturally- competent nursing. Community nursing does not necessarily require that a nurse become more assertive with client. It would be simplistic, and in most cases inappropriate, to guide clients to replace biomedical interventions or avoid doctors. 7. In spite of the important role that hospitals play in American healthcare, there is growing importance of community-based healthcare and community-based nursing. Which of the following statements best conveys a central aspect of the philosophy of community care? A) The client is in charge of his or her health and healthcare in the community. B) Nurses maximize their scope of practice in noninstitutional settings. C) Community settings allow for the greatest number and variety of treatment options. D) The nurse becomes the key member of the healthcare team in a community setting. Ans: A Feedback: A central premise of community healthcare is the fact that patients/clients are in charge. The move toward community care is not motivated by an increased role for nurses. There are a greater number of treatment options in hospitals than in the community, but this fact does not negate the importance of community care. 8. Mr. Hammond is a 70-year-old man with a diagnosis of type 1 diabetes who developed a diabetic foot ulcer earlier this year. He has recently been discharged from the hospital and now requires regular wound care. Karen is a community health nurse who provides wound care for Mr. Hammond twice weekly. Which of Karens actions is most likely to empower Mr. Hammond? A) Encourage Mr. Hammond to acknowledge his contribution to the development of his wound. B) Provide information to Mr. Hammond that matches his expressed needs. C) Encourage Mr. Hammond to involve members of his family in his care. D) Delegate wound care to Mr. Hammond and reduce the frequency of her visits. Ans: B Feedback: Client empowerment is often fostered by assessing and meeting a clients need for information. Encouraging an acknowledgement of the clients contribution to his or her current health state is beneficial in many circumstances, but it less likely to make the client feel empowered. Similarly, family involvement can be beneficial but does not directly foster empowerment. Empowerment does not necessarily mean that the nurse does less and the client performs his or her own care; delegation may not be appropriate. 9. An elderly female client who resides in the community tends to defer decisions regarding her care to her eldest son. How should the community health nurse respond to the clients reluctance to make independent decisions? A) Discuss this observation with the client and her son in an open manner and explore alternatives. B) Organize care so that it takes place at times when the son is not present in the home. C) Accommodate this aspect of the clients family dynamics when planning and carrying out care. D) Teach the client assertiveness skills that she can apply in her interactions with her son. Ans: C Feedback: The nurse should respect the clients desire to organize her care in the way that she prefers. It is not the responsibility of the nurse to reorganize or overcome this familys dynamics. 10. A client with a long-standing diagnosis of chronic obstructive pulmonary disease (COPD) has been enrolled in a disease management program. Which of the following activities will be prioritized in this program? A) Providing comprehensive and evidence-based care of the clients COPD B) Creating collaborative relationships between the client and the care team C) Ensuring that the client qualifies for Medicare and Medicaid D) Liaising between the client and his health maintenance organization (HMO) Ans: A Feedback: Disease management focuses on providing the best evidence-based care for an individual with a specific chronic illness. This does not necessitate enrollment in an HMO, Medicare, or Medicaid. Collaborative relationships facilitate effective disease management but this is a means to the end of positive health outcomes rather than an end in itself. 11. One of the expressed goals of Healthy People 2020 is to achieve health equity and eliminate disparities. What health indicator can most accurately gauge whether this goal is being achieved? A) Environmental quality B) Injury and violence C) Mental health D) Access to healthcare services 12. Nurses have the potential to positively impact the health of communities. Which of the following actions is most likely to improve the health of a community? A) Publicizing the consequences of unhealthy lifestyles B) Advocating politically for laws and policies that foster community health C) Ensuring that nurses are practicing to the full extent of their scope of practice D) Providing nursing care to individuals who are not patients or clients 13. A nurse who provides care in an acute medical unit is aware of the importance of thorough discharge planning. The discharge planning process should begin A) once the patient has stabilized and is assured of positive outcomes. B) as soon as possible after the patient is admitted. C) once the patient has received a discharge order from his or her primary care provider. D) 48 to 72 hours before the projected date of discharge. 14. A hospital patient has discussed with the nurse her use of visualization, biofeedback, and relaxation exercises in managing the chronic pain that results from her fibromyalgia. The nurse should recognize this patients use of what category of complementary/alternative medicine (CAM)? A) Biologically-based practices B) Manipulative practices C) Traditional indigenous medicine D) Mind-body medicine 15. A hospital patient who suffered a spinal cord injury has expressed an interest in exploring complementary/alternative therapies. The nurse should encourage the patient to begin this process by doing which of the following activities? A) Asking practitioners of different therapies to provide lists of satisfied clients B) Asking the patients primary care provider for permission to explore nonbiomedical treatments C) Finding reliable evidence regarding the safety and effectiveness of therapies D) Determining whether the patients health insurance would cover the cost of alternative/complementary therapies Chapter 6. Common Neurological Complaints Multiple Choice Identify the choice that best completes the statement or answers the question. 1. Which statement about confusion is true? a. Confusion is a disease process. b. Confusion is always temporary. c. Age is a reliable predictor of confusion. d. Polypharmacy is a major contributor to confusion in older adults. 2. Sondra’s peripheral vestibular disease causes dizziness and vertigo. Which of the following medications will help to decrease edema in the labyrinth of the ear? a. Meclizine b. Diphenhydramine c. Diamox d. Promethazine 3. The hallmark of an absence seizure is: a. No activity at all b. A blank stare c. Urine is usually voided involuntarily d. The attack usually lasts several minutes 4. How often should drug levels be monitored when a seizure medication has controlled the seizures, and the drug level is adequate? a. Every 3 months b. Every 6 months c. Annually d. Whenever there is a problem 5. Which of the following persons fits the classic description of a patient with multiple sclerosis (MS)? a. A teenage male b. A 65-year-old male c. A 25-year-old female d. A 60-year-old female 6. Which of the following is a specific test to MS? a. Magnetic resonance imaging (MRI) b. Computed tomography (CT) scan c. A lumbar puncture d. There is no specific test. 7. Which drug for Alzheimer’s disease should be administered beginning at the time of diagnosis? a. Cholinesterase inhibitors b. Anxiolytics c. Antidepressants d. Atypical antipsychotics 8. Which hematoma occurs along the temporal cranial wall and results from tears in the middle meningeal artery? a. Epidural hematoma b. Subdural hematoma c. Subarachnoid hematoma d. Intraparenchymal hemorrhage 9. Which cranial nerve is affected in a patient with a cerebrovascular accident who has difficulty chewing? a. CN V b. CN VII c. CN IX d. CN X 10. Which statement best describes a carotid bruit? a. It is felt with the middle three fingers over the carotid artery. b. A bruit becomes audible when the lumen is narrowed to 1 mm or less. c. A low-pitched bruit is a medical emergency. d. The higher the pitch of the bruit, the higher the degree of stenosis. 11. Which patient is more likely to have a cluster headache? a. A female in her reproductive years b. A 40-year-old African American male c. A 55-year-old female who drinks 10 cups of coffee daily d. A 45-year-old male awakened at night 12. Inattention and a sleep-wake cycle disturbance are the hallmark symptoms of? a. Dementia b. Alzheimer’s disease c. Parkinson’s disease d. Delirium 13. Which type of meningitis is more benign, self-limiting, and caused primarily by a virus? a. Purulent meningitis b. Chronic meningitis c. Aseptic meningitis d. Herpes meningitis 14. Which is the most sensitive neuroimaging test to evaluate patients with encephalitis? a. MRI b. CT c. Electroencephalogram (EEG) d. An initial lumbar puncture 15. What is usually the first sign or symptom that a patient would present with that would make you suspect herpes zoster? a. A stabbing pain on one small area of the body b. A vesicular skin lesion on one side of the body c. A pain that is worse upon awakening d. A lesion on the exterior ear canal 16. Gabby, aged 22, has Bell’s palsy on the right side of her face. Her mouth is distorted, and she is concerned about permanent paralysis and pain. What do you tell her? a. “Most patients have complete recovery in 3 to 6 months.” b. “Unfortunately, you’ll probably have a small amount of residual damage.” c. “Don’t worry, I’ll take care of everything.” d. “You may have a few more episodes over the course of your lifetime but no permanent damage.” 17. Sam, aged 65, is started on L-dopa for his Parkinson’s disease (PD). He asks why this is necessary. You tell him: a. “L-dopa is neuroprotective.” b. “The primary goal of therapy is to replace depleted stores of dopamine.” c. “This is the only drug that can provide symptomatic benefit.” d. “This is the initial monotherapy drug.” 18. Which of the following signs is seen in a patient with more advanced PD? a. Resting tremor b. Bradykinesia c. Rigidity d. Postural instability 19. Which of the following is the most commonly experienced symptom of migraine? a. Light sensitivity b. Pulsatile pain c. Sound sensitivity d. Experiencing an aura 20. Which of the following characteristics differentiates peripheral vertigo from central vertigo? a. The duration of central vertigo is shorter than that of peripheral vertigo. b. There is an auditory-associated symptom with peripheral vertigo and a visual- associated symptom with central vertigo. c. Central vertigo is positional, and peripheral vertigo is not. d. The onset of central vertigo is more sudden than that of peripheral vertigo. 21. Carotid endarterectomy should be considered only for symptomatic patients with greater than what percentage of stenosis? a. Greater than 25% b. Greater than 50% c. Greater than 75% d. Only for 100% occlusion 22. What antiplatelet agent is most widely used for secondary prevention of stroke? a. Aspirin b. Ticlopidine c. Clopidogrel d. Aspirin and clopidogrel 23. Which adjunctive diagnostic test should be used in the work-up of a patient with suspected Creutzfeldt-Jakob disease or transient epileptic amnesia? a. MRI b. CT c. Cerebrospinal fluid analysis d. EEG 24. Which herbal preparation may cause delirium and should be avoided in an elderly patient? a. Sam-e b. Saint John’s Wort c. Melatonin d. Saw Palmetto 25. Which of the following activities is part of the functional activities questionnaire? a. Asking the patient to unravel a Rubik’s cube b. Determining if the patient can drive on the highway c. Asking the patient about a news event from the current week d. Seeing if the patient can keep his or her home clean 26. About 90% of all headaches are? a. Tension b. Migraine c. Cluster d. Without pathological cause 27. Which statement is true regarding driving and patients with a seizure disorder? a. Once diagnosed with a seizure disorder, patients must never drive again. b. After being seizure free for 6 months, patients may drive. c. Each state has different laws governing driving for individuals with a seizure disorder. d. These persons may drive but never alone. 28. Julie has relapsing-remitting muscular sclerosis. She has not had a good response to interferon. Which medication might help given intravenously once a month? a. Glatiramer acetate b. Natalizumab c. Fingolimod d. Glucocorticoids 29. The ‘freezing phenomenon’ is a cardinal feature of? a. Parkinson’s disease b. Alzheimer’s disease c. A CVA d. Bell’s palsy 30. A ratchet-like rhythmic contraction, especially in the hand, during passive stretching is known as? a. Spinothalamic dysfunction b. Ratcheting c. Cogwheeling d. Hand tremors 31. Clinical features of insidious onset, slow progression, and a lack of other findings to explain the symptoms are fairly diagnostic of which condition? a. Guillain-Barré syndrome b. Parkinson’s disease c. Alzheimer’s disease d. Huntington’s disease 32. Which condition is characterized by the impaired ability to learn new information along with either a cognitive disturbance in language, function, or perception? a. Guillain-Barré syndrome b. Parkinson’s disease c. Alzheimer’s disease d. Delirium 33. A score of 20 to 25 on this test indicates early-stage Alzheimer’s disease: a. SLUMS b. MoCA c. FAST d. MMSE 34. Intravenous thrombolytic therapy following an ischemic CVA should be given within how many hours of symptom onset? a. 1 hour b. 3 hours c. 6 hours d. 12 hours 35. When administered at the beginning of an attack, oxygen therapy may help this kind of headache? a. Tension b. Migraine c. Cluster d. Stress Common Skin Complaints Multiple Choice Identify the choice that best completes the statement or answers the question. 1. Simon presents with alopecia areata with well-circumscribed patches of hair loss on the crown of his head. How do you respond when he asks you the cause? a. “You must be under a lot of stress lately.” b. “It is hereditary. Did your father experience this also?” c. “The cause is unknown, but we suspect it is due to an immunologic mechanism.” d. “We’ll have to do some tests.” 2. Which of the following is “a linear crack extending from the epidermis to the dermis?” a. An ulcer b. A fissure c. Lichenification d. An excoriation 3. A bulla is: a. A vesicle larger than 1 cm in diameter b. An elevated solid mass with a hard texture; the shape and borders can be regular or irregular c. A superficial elevated lesion filled with purulent fluid d. Thinning of the skin (epidermis and dermis) that appears white or translucent 4. An example of ecchymosis is: a. A hematoma b. A keloid c. A bruise d. A patch 5. When looking under the microscope to diagnose an intravaginal infection, you see a cluster of small and oval to round shapes. What do you suspect they are? a. Spores b. Leukocytes c. Pseudohyphae d. Epithelial cells 6. Your patient is in her second trimester of pregnancy and has a yeast infection. Which of the following is a treatment that you usually recommend/order in nonpregnant patients, but is listed as a Pregnancy category D? a. Vagistat vaginal cream b. Monistat combination pack c. Terazol vaginal cream d. Diflucan, 150 mg 7. Tinea unguium is also known as: a. Onychomycosis b. Tinea versicolor c. Tinea manuum d. Tinea corporis 8. Sally, age 25, presents with impetigo that has been diagnosed as infected with Staphylococcus. The clinical presentation is pruritic tender, red vesicles surrounded by erythema with a rash that is ulcerating. Her recent treatment has not been adequate. Which type of impetigo is this? a. Bullous impetigo b. Staphylococcal scalded skin syndrome (SSSS) c. Nonbullous impetigo d. Ecthyma 9. Mark has necrotizing fasciitis of his left lower extremity. Pressure on the skin reveals crepitus due to gas production by which anaerobic bacteria? a. Staphylococcal aureus b. Clostridium perfringens c. S. pyrogenes d. Streptococcus 10. When using the microscope for an intravaginal infection, you see something translucent and colorless. What do you suspect? a. A piece of hair or a thread b. Hyphae c. Leukocytes d. Spores 11. Marci has a wart on her hand. She says she heard something about “silver duct tape therapy.” What do you tell her about his? a. It is an old wives’ tale. b. It is used as a last resort. c. Salicylic acid is more effective. d. It is a simple treatment that should be tried first. 12. Which is the most potent and irritating dose of tretinoin? a. 0.05% liquid formulation b. 0.1% cream c. 1% foam d. 0.02% cream 13. Of the following types of cellulitis, which is a streptococcal infection of the superficial layers of the skin that does not involve the subcutaneous layers? a. Necrotizing fasciitis b. Periorbital cellulitis c. Erysipelas d. “Flesh-eating” cellulitis 14. Mandy presents with a cauliflower-like wart in her anogenital region. You suspect it was sexually transmitted and document this as a: a. Filiform/digitate wart b. Dysplastic cervical lesion c. Condyloma acuminata d. Koilocytosis 15. Jeffrey has atopic dermatitis. You are prescribing a low-dose topical corticosteroid for him. Which would be a good choice? a. Betamethasone dipropionate 0.05% b. Hydrocortisone base 2.5% c. Halcinonide 0.1% d. Desonide 0.05% 16. Harvey has a rubbery, smooth, round mass on his chest that is compressible and has a soft-to-very- firm texture. What do you diagnose this as? a. A lipoma b. A nevi c. A skin tag d. A possible adenoma 17. Which of the following statements is accurate when you are removing a seborrheic keratosis lesion using liquid nitrogen? a. Do not use lidocaine as it may potentiate bleeding. b. Pinch the skin taut together. c. Use gel foam to control bleeding. d. This should be performed by a dermatologist only. 18. The “B” in the ABCDEs of assessing skin cancer represents: a. Biopsy b. Best practice c. Boundary d. Border irregularity 19. The majority of HSV-1 and HSV-2 infections are asymptomatic so that only which elevated antibody titer shows evidence of previous infection? a. IgA b. IgE c. IgG d. IgM 20. Eighty percent of men have noticeable hair loss by what age? a. 35 b. 50 c. 70 d. 85 21. Prevalence of psoriasis is highest in which group? a. Scandinavians b. African Americans c. Asians d. Native Americans 22. The most common precancerous skin lesion found in Caucasians is: a. A skin tag b. Actinic keratosis c. A melanoma d. A basal cell lesion 23. Ian, age 62, presents with a wide, diffuse area of erythematous skin on his lower left leg that is warm and tender to palpation. There is some edema involved. You suspect: a. Necrotizing fasciitis b. Kaposi’s sarcoma c. Cellulitis d. A diabetic ulcer 24. Josh, aged 22, has tinea versicolor. Which description is the most likely for this condition? a. There are round, hypopigmented macules on his back. b. Josh has red papules on his face. c. There are crusted plaques in Josh’s groin area. d. There are white streaks on his neck. 25. Tori is on systemic antifungals for a bad tinea infection. You are aware that the antifungals may cause: a. Renal failure b. Skin discoloration c. Breathing difficulties d. Hepatotoxicity 26. Which scalp problem can be caused by a fever and certain drugs? a. Telogen effluvium (TE) b. Trichotillomania c. Psoriasis d. Alopecia areata 27. Why do people of African descent have a lower incidence of non-melanoma skin cancer? a. They have an increased number of melanocytes. b. Their darker skin protects from ultraviolet radiation. c. Their skin is thicker. d. Their immune system is stronger. 28. Which statement is true regarding chloasma, the ‘mask of pregnancy’? a. It is caused by a decrease in the melanocyte-stimulating hormone during pregnancy. b. This condition only occurs on the face. c. Exposure to sunlight will even out the discoloration. d. It is caused by increased levels of estrogen and progesterone. 29. When instructing your elderly client about treating her xerosis, what do you tell her? a. A daily hot bath may help the associated pruritus. b. Rub the skin briskly to make sure it is completely dry after bathing. c. Only take short tepid showers. d. Use a gel that is alcohol-based after bathing to soften the skin. 30. Which medication used for scabies is safe for children 2 months and older? a. Permethrin cream b. Lindane c. Crotamiton lotion and cream d. Ivermectin 31. Which of the following is an infraorbital fold skin manifestation in a patient with atopic dermatitis? a. Keratosis pilaris b. Dennie’s sign c. Keratoconus d. Pityriasis alba 32. Which of the following statements about performing cryosurgery for actinic keratosis is true? a. It is better to slightly overfreeze the area, so you only have to do it once. b. Using liquid nitrogen, freeze each lesion for at least 30 seconds. c. Every lesion should be biopsied after using liquid nitrogen. d. The ‘freeze balls’ should be approximately one-and-a-half times as wide as they are deep. 33. An example of a primary skin lesion is a/an: a. Bulla b. Scale c. Excoriation d. Fissure 34. Which statement regarding necrotizing fasciitis is true? a. The hallmark of this infection is its slow and steady progression. b. Once the border of the infection is “established,” it does not spread. c. Loss of life or limb is a potential complication. d. The lesion is most dangerous, because it is painless. 35. When staging a malignant melanoma using Clark’s levels, which level extends into the papillary dermis? a. Level I b. Level II c. Level III d. Level IV Parasitic Skin Infestations 1. The school nurse recognizes the signs of scabies when a child presents with: a. small fluid filled blisters that sting when scratched. b. dry scaly patches in body creases that itch. c. wavy threadlike lines on the body and pruritus. d. cluster of papular lesions with pruritus. 2. Which of the following are nursing interventions and patient teaching for the treatment of head lice and scabies? (Select all that apply.) a. Clothing, linens, and bath articles thoroughly cleaned in hot water b. Stress nature and transmission of the disease c. Special carbohydrate diet to promote healing d. Complete isolation from the public 3. The nurse is caring for a patient with lesions on the skin. Which assessment finding should cause the nurse to suspect scabies? a. Large, fluid-filled blisters b. Short, wavy, brownish black lines c. Reddish brown dots at the base of hairs d. Gray blue macules on the thighs and axillae Common Cardiovascular Complaints Multiple Choice Identify the choice that best completes the statement or answers the question. 1. Which group would most benefit from statins? a. Those with a low density lipoprotein-cholesterol greater than 100 mg/dL b. Individuals with clinical arteriosclerotic cardiovascular disease c. Individuals with a 10-year risk greater than 10% d. Individuals of all ages with diabetes mellitus (DM) 2. If chest pain can be alleviated with time, analgesics, and heat applications, what might the differential diagnosis be? a. Peptic ulcer b. Hiatal hernia c. Costochondritis d. Pericarditis 3. Sandra has palpitations that occur with muscle twitching, paresthesia, and fatigue. What specific diagnostic test might help determine the cause? a. Serum calcium b. Electrocardiogram (ECG) c. Thyroid-stimulating hormone test d. Complete blood cell count 4. A blood pressure (BP) of 150/90 is considered: a. Stage 2 hypertension b. Hypertensive c. Normal in healthy older adults d. Acceptable if the patient has DM 5. Lifestyle modifications to manage hypertension (HTN) include: a. Maintaining a body mass index of 17 b. Restricting dietary sodium to 2 grams per day c. Engaging in exercise or physical activity for 90 minutes a day d. Limiting beer intake to 24 ounces per day 6. Mary has hypertension and previously had a stroke. Which hypertensive drug would you order for her? a. Angiotensin converting enzyme inhibitor b. Calcium channel blocker c. Angiotensin II receptor blocker d. Beta blocker 7. Which high-density lipoprotein (HDL) level is considered cardioprotective? a. Greater than 30 b. Greater than 40 c. Greater than 50 d. Greater than 60 8. You are assessing Sigred for metabolic syndrome. Which of her parameters is indicative of this syndrome? a. Her waist is 36 inches. b. Her triglyceride level is 140 mg/dL. c. Her BP is 128/84. d. Her fasting blood sugar (BS) is 108 mg/dL. 9. Which type of angina do you suspect in Harvey, who complains of chest pain that occurs during sleep and most often in the early morning hours? a. Stable angina b. Unstable angina c. Variant (Prinzmetal’s angina) d. Probably not angina but hiatal hernia 10. Which ECG change is typical of cardiac ischemia? a. T-wave inversion b. ST-segment elevation c. Significant Q wave d. U-wave 11. In which type of arterioventricular (AV) block does the pulse rate (PR) interval lengthen until a beat is dropped? a. First-degree AV block b. Second-degree Mobitz I AV block c. Second-degree Mobitz II AV block d. Third-degree AV block 12. A Delta wave on the ECG may be present in which condition? a. Prinzmetal’s angina b. Bundle branch block c. Wolff-Parkinson-White syndrome d. Aortic stenosis 13. Which heart sound may be heard with poorly controlled hypertension, angina, and ischemic heart disease? a. A physiologic split S2 b. A fixed split S2 c. S3 d. S4 14. Samuel is going to the dentist for some work and must take endocarditis prophylaxis because of his history of: a. Severe asthma b. A common valvular lesion c. Severe hypertension d. A prosthetic heart valve 15. George, age 64, has cardiovascular disease (CVD), a total cholesterol of 280 mg/dL, and a systolic BP of 158. He is being treated for hypertension. You are doing a Framingham Risk Assessment on him. Which assessment factor would give him the highest number of points on the scale? a. His age b. His cholesterol level c. His systolic BP d. The fact that he is on antihypertensive medication 16. Which pain characteristic is usually indicative of cardiac pathology? a. Fleeting b. Moving c. Diffuse d. Localized 17. What percentage of patients with angina pectoris will have simultaneous dyspnea, caused by transient increase in pulmonary venous pressures that accompany ventricular stiffening during an episode of myocardial ischemia? a. About 20% b. About 30% c. About 50% d. Almost all 18. Nitroglycerine (NTG) is given for a patient having ischemic chest pain. One tablet or one spray should be used under the tongue every 5 minutes for three doses. What should be done if the pain has not been relieved after three doses? a. 911 should be called, and the patient should be transported immediately to the emergency department. b. One more dose of NTG may be tried. c. The person should be given two aspirin to chew. d. A portable defibrillator should be located to ascertain the cardiac rhythm. 19. For the best therapeutic effect after a myocardial infarction (MI), thrombolytics should be administered within the first 3 hours (ideally 30 minutes) of symptom onset. Studies have shown, however, that thrombolytic therapy can be of benefit up to how many hours after the initial presentation of MI symptoms? a. 6 hours b. 8 hours c. 10 hours d. 12 hours 20. When teaching post MI patients about their NTG tablets, the clinician should stress that the tablets should remain in the light-resistant bottle in which they are packaged and should not be put in another pill box or remain in areas that are or could become warm and humid. Once opened, the bottle must be dated and discarded after how many months? a. 1 month b. 3 months c. 6 months d. As long as the tablets are kept in this special bottle, they will last forever. 21. There are four stages of heart failure, classified as A to D, that describe the evolution and progression of disease. In which stage are patients hospitalized or treated with specialized interventions or hospice care for refractory symptoms of heart failure despite medical therapy? a. Stage A b. Stage B c. Stage C d. Stage D 22. Which of the following is abundant in the heart and rapidly rises in the bloodstream in the presence of heart failure, making it a good diagnostic test? a. B-type natriuretic peptide b. C-reactive protein c. Serum albumin d. Erythrocyte sedimentation rate 23. Which test has long been considered the gold standard for a diagnosis of venous thromboembolism? a. Ultrasound b. Magnetic resonance imaging (MRI) c. Ascending venogram d. D-dimer 24. Statins are approved for which age group? a. Children over the age of 2 b. Children over the age of 6 c. Children over the age of 10 d. Only adolescents and adults 25. The American College of Cardiology/American Heart Association states which of the following regarding the use of non-statin lipid-lowering agents? a. Nicotinic acid derivatives are effective for lowering LDL and triglycerides (TGs). b. Bile acid sequestrates increase HDL. c. Cholesterol absorption inhibitors decrease LDL. d. There is no sufficient evidence to use non-statin lipid-drugs. 26. Which of the following medications can cause hyperlipidemia? a. Diuretics b. NSAIDs c. Opioids d. Insulin 27. Jamie, age 55, has just started on a statin after having his liver function tests (LFTs) come back normal. He now asks you how often he has to have the LFTs repeated. What do you tell him? a. Initially in 6 weeks b. Every 3 months c. Every 6 months d. It’s no longer necessary for his statin regimen. 28. In the CHADS2 Index for the stroke risk score for AF, the ‘A’ stands for: a. Anticoagulation b. Autoimmune disease c. Age d. Antihypertension 29. Which murmurs are usually ‘watch and wait’? a. Systolic murmurs b. Diastolic murmurs c. They both are dangerous and need immediate attention. d. You can ‘watch and wait’ for both of them. 30. Which of the following statements about dabigatran is true? a. It is difficult to keep the patient in therapeutic range. b. Anticoagulation cannot be immediately reversed. c. It allows for the use of tPA if the patient has a stroke despite anticoagulation. d. None of the statements are true. 31. What value on the ankle-brachial index diagnoses peripheral artery disease? a. Less than 0.25 b. Less than 0.50 c. Less than 0.90 d. Greater than 1 32. Your patient with permanent afib asks when he can discontinue his warfarin. You tell him: a. When your internalized normalized ratio reaches 3.0, you can stop taking your warfarin permanently. b. When you no longer feel ill c. One month after your symptoms dissipate d. You’ll probably be on it indefinitely. 33. You just started Martha on HTN therapy. The Eighth Joint National Committee recommends that if her goal BP is not reached in what length of time, you should increase the initial drug or add a second drug to it? a. 1 month b. 3 months c. 6 months d. 1 year Common Abdominal Complaints Multiple Choice Identify the choice that best completes the statement or answers the question. 1. A 35-year-old female patient is seen in the clinic complaining of abdominal pain. Which of the following should be included in the history and physical examination? a. Digital rectal exam b. Pelvic exam c. Sexual history d. All of the above 2. A patient comes to the office complaining of constipation. The patient lists all of the following medications. Which drug could be responsible for the constipation? a. Multivitamin b. Magnesium hydroxide c. Pepto-Bismol® d. Ibuprofen 3. A patient is seen with complaints of diarrhea. Which of the following should be included in the patient’s differential diagnosis? a. Gastroenteritis b. Inflammatory bowel disease c. Lactase deficiency d. All of the above 4. Mr. J. K., 38 years old, is 5 feet 8 inches tall and weighs 189 pounds. He reports that he has had intermittent heartburn for several months and takes Tums® with temporary relief. He has been waking during the night with a burning sensation in his chest. Which additional information would lead you to believe that gastroesophageal reflux disease (GERD) is the cause of his pain? a. The pain seems better when he smokes to relieve his nerves. b. Coffee and fried foods don’t bother him, c. He wakes at night coughing with a bad taste in his mouth. d. All of the above 5. A 29-year-old Englishman is seen in the office with complaints of pain in his chest and belly. He has been suffering the pain for 2 weeks and gets temporary relief from Alka-Seltzer®. The burning pain wakes him at night and radiates up to his chest. Which factor favors a diagnosis of gastric ulcer? a. His gender b. His age c. His use of Alka-Seltzer d. His ethnic origin 6. Which of the following is most effective in diagnosing appendicitis? a. History and physical b. Sedimentation rate c. Kidney, ureter, and bladder x-ray d. Complete blood count (CBC) with differentials 7. Which of the following is associated with celiac disease (celiac sprue)? a. Malabsorption b. Constipation c. Rectal bleeding d. Esophageal ulceration 8. A 45-year-old patient presents with a chief complaint of generalized abdominal pain. Her physical examination is remarkable for left lower quadrant tenderness. At this time, which of the following should be considered in the differential diagnosis? a. Endometriosis b. Colon cancer c. Diverticulitis d. All of the above 9. A 46-year-old patient is seen in the clinic with abdominal pain. Which of the following tests is essential for this patient? a. CBC with differential b. Urine human chorionic gonadotropin c. Barium enema d. Computed tomography of the abdomen 10. A 25-year-old accountant is seen in the clinic complaining of crampy abdominal pain after meals. She is often constipated and takes laxatives, which are followed by a couple of days of diarrhea. She temporarily feels better after a bowel movement. She states she is embarrassed by flatulence and has abdominal distension. She has had no weight loss or blood in her stool. This problem has gone on for about 6 months. What should the next step be? a. Obtain a complete history. b. Order a barium enema. c. Schedule a Bernstein’s test. d. Prescribe a trial of antispasmodics. 11. A 28-year-old patient is seen in the clinic with colicky abdominal pain particular with meals. She has frequent constipation, flatulence, and abdominal distension. Which of the data make a diagnosis of diverticulitis unlikely? a. Her age b. Frequent constipation c. Flatulence d. Colicky abdominal pain 12. A 28-year-old patient is seen with complaints of diarrhea. Which of the following responses to the history questions would help the primary care physician (PCP) establish the diagnosis of irritable bowel syndrome? a. Feels relief after a bowel movement b. Sometimes is constipated c. Does not defecate in the middle of the night d. All of the above 13. A patient is diagnosed with GERD, and his endoscopic report reveals the presence of Barrett’s epithelium. Which of the following should the PCP include in the explanation of the pathology report? a. This is a premalignant tissue. b. This tissue is resistant to gastric acid. c. This tissue supports healing of the esophagus. d. All of the above 14. Which of the following dietary instructions should be given to a patient with GERD? a. Eliminate coffee. b. Drink peppermint tea to relieve stomach distress. c. Recline and rest after meals. d. Limit the amount of antacids. 15. The patient with GERD should be instructed to eliminate which of these activities? a. Swimming b. Weight lifting c. Golfing d. Walking 16. A patient is diagnosed with giardia after a backpacking trip in the mountains. Which of the following would be an appropriate treatment? a. Vancomycin b. Penicillin c. Metronidazole d. Bactrim 17. A 22-year-old is seen complaining of vague belly pain. This type of pain is seen at what point in appendicitis? a. Very early b. 3 to 4 hours after perforation c. Late in inflammation d. Appendicitis never presents with vague pain. 18. The nurse practitioner (NP) suspects a patient has a peptic ulcer. Which of the following items on the history would lead the NP to this conclusion? a. Use of NSAIDs b. Cigarette smoker c. Ethanol consumption d. All of the above 19. A patient is seen with dark-colored urine, and the urine dipstick reveals a high level of bilirubin. Which of the following could be a cause of this problem? a. Increased breakdown of red blood cells b. Inadequate hepatocyte function c. Biliary obstruction d. All of the above 20. A 21-year-old student presents with complaints of fatigue, headache, anorexia, and a runny nose, all of which began about 2 weeks ago. She started taking vitamins and over-the-counter cold preparations but feels worse. The smell of food makes her nauseated. Her boyfriend had mononucleosis about a month ago, and she wonders if she might have it also. Examination reveals cervical adenopathy and an enlarged liver and spleen. Which of the following labs would be most helpful in the differential diagnosis at this point? a. Stool culture b. Liver enzymes c. Antihepatitis D virus d. Thyroid-stimulating hormone test 21. On further questioning, the 21-year-old patient with complaints of fatigue, headache, anorexia, and a runny nose explains that she is sexually active only with her boyfriend, does not use injectable drugs, and works as an aide in a day-care center. Which of the following tests would be most helpful in confirming your diagnosis? a. Hepatitis A virus (HAV) IgM b. HAV IgG c. Anti-HAcAg d. Anti-HAsAg 22. A patient is seen in the clinic with right upper quadrant pain that is radiating to the middle of the back. The NP suspects acute cholelithiasis. The NP should expect which of the following laboratory findings? a. Decreased alanine aminotransferase and decreased aspartate aminotransferase b. Elevated alkaline phosphatase c. Elevated indirect bilirubin d. Decreased white blood cells 23. A patient has acute pancreatitis with seven of the diagnostic criteria from Ranson’s criteria. In order to plan care, the NP must understand that this criteria score has which of the following meanings? a. A high mortality rate b. An increased chance of recurrence c. A 7% chance of the disease becoming chronic d. All of the above 24. A patient is seen in the office with complaints of six to seven liquid bowel movements per day. Which of the following assessment findings would lead the NP to a diagnosis of inflammatory bowel disease? a. Intermittent constipation with periods of diarrhea b. Wakens at night with diarrhea c. History of international travel d. All of the above 25. Which of the following is part of the treatment plan for the patient with irritable bowel syndrome? a. High fiber diet b. Tylenol with codeine c. Daily laxatives d. All of the above Vaginal, Uterine, and Ovarian Disorders Multiple Choice Identify the choice that best completes the statement or answers the question. 1. A 23-year-old sexually active woman presents for her first Pap smear. Her history includes nulligravida, age at first intercourse 14, and more than 10 sexual partners. Which of the following conditions should the clinician be particularly alert for during her examination? a. Human papillomavirus (HPV) b. Endometrial hyperplasia c. Vagismus d. Polycystic ovarian syndrome 2. A 20-year-old woman is seen in the clinic because her boyfriend was found to have gonorrhea. Which of the following is the treatment of choice for gonorrhea? a. Ceftriaxone b. Doxycycline c. Acyclovir d. Metronidazole 3. A 24-year-old woman presents to the clinic with dysuria, dyspareunia, and a mucopurulent vaginal discharge. Her boyfriend was recently treated for nongonococcal urethritis. What sexually transmitted disease has she most probably been exposed to? a. Gonorrhea b. HPV c. Chlamydia d. Trichomonas 4. A 45-year-old woman is seen in the clinic with complaints of a vaginal discharge. The clinician identifies clue cells on the vaginal smear. Which of the following diagnoses is associated with this finding? a. Trichomonas b. Bacterial vaginosis c. HPV d. Herpes simplex virus 5. Which of the following medications is the treatment of choice for trichomonas? a. Metronidazole b. Ceftriaxone c. Diflucan d. Doxycycline 6. A 58-year-old woman presents with a breast mass. Which of the following responses by the clinician would be most appropriate? a. “It is probably just a cyst because that is the most common breast mass.” b. “We will order a mammogram and ultrasound to help establish a diagnosis.” c. “We will go ahead and schedule you for a biopsy because that is the only way to know for sure.” d. “Because your lump is painful, it is most likely not cancer.” 7. A 26-year-old woman is seen with complaints of irregular vaginal bleeding. Which of the following tests should be the first priority? a. Pregnancy test b. Pelvic ultrasound c. Endometrial biopsy d. Platelet count 8. A 42-year-old woman presents to the clinic with complaints of painful intercourse for the last month. Which of the following should be explored as the likely cause of her dyspareunia? a. Menopause b. Dehydration c. Excess progesterone d. Sexual trauma as a child 9. A 36-year-old woman is seen with complaints of vaginal itching, burning, and discharge. On potassium hydroxide (KOH) wet mount of vaginal discharge, the clinician notices hyphae. Which of the following treatments would be appropriate? a. Fluconazole b. Estrogen vaginal cream c. Metronidazole d. Doxycycline 10. A 21-year-old woman is seen in the clinic requesting birth control pills. Which of the following tests is essential before prescribing any oral contraceptive? a. Pregnancy test b. Complete blood cell count c. Thyroid-stimulating hormone d. Urine dip for protein 11. A 40-year-old woman is seen for her yearly examination. She is single and not in a monogamous relationship. Her social history includes smoking cigarettes “occasionally” and drinking about two beers a day. Her body mass index (BMI) is 25. She is requesting birth control. Which of the following methods would be best for this patient? a. Intrauterine device b. Oral contraceptive c. Condom d. Vaginal contraceptive sponge 12. A 44-year-old patient with breast cancer is prescribed tamoxifen by her surgeon. She is complaining about hot flashes. Which of the following responses by the clinician would be most appropriate? a. “You must be having menopause.” b. “The hot flashes are a result of the antiestrogenic effects of tamoxifen.” c. “Tamoxifen will impact your temperature regulation center.” d. “The drug destroys your ovaries.” 13. A 32-year-old woman is seen in the clinic because she has been unable to get pregnant after 12 months of unprotected sex. In order to determine the cause of the infertility, the clinician should question her about which of these possible causes? a. Pelvic inflammatory disease b. Oral contraceptive use for 15 years c. Early menarche d. Diet high in soy protein 14. When assessing a woman for infertility, which of the following tests should be done first? a. Analysis of partner’s sperm b. Magnetic resonance imaging (MRI) c. Hysterosalpingogram d. Estrogen level 15. A 15-year-old girl is seen in the clinic because she has not yet had her first period. Which of the following questions would help the clinician determine the cause? a. “Are you sexually active?” b. “How long have you been underweight?” c. “Did your mother take diethylstilbestrol during her pregnancy?” d. “Have you noticed any changes in your moods lately?” 16. What is the most common cause of secondary amenorrhea? a. Pregnancy b. Pituitary dysfunction c. Inadequate estrogen levels d. Genetic disorders 17. A 22-year-old woman is diagnosed with premenstrual syndrome. Which of the following lifestyle changes should the clinician suggest to help minimize the patient’s symptoms? a. At least 4 cups of green tea daily b. Regular exercise c. Take vitamin A supplements d. Eat a diet high in iron 18. A 25-year-old woman is seen in the clinic complaining of painful menstruation. Which of the following pelvic pathologies is the most common cause of dysmenorrhea? a. Pelvic inflammatory disease b. Endometriosis c. Sexually transmitted infections d. Ovarian cyst 19. A 26-year-old woman tells the clinician that she has endometriosis, because she has frequent pelvic pain. The clinician also should consider which of these differential diagnoses? a. Diverticulitis b. Cholelithiasis c. Kidney stones d. Ovarian cysts 20. Which of the following would be appropriate treatment for a woman with mild endometriosis? a. Oral contraceptives b. Leuprolide acetate injections c. Nafarelin nasal spray d. Hysterectomy 21. A 45-year-old woman is seen in the clinic with abnormal uterine bleeding and pain during intercourse. The clinician should consider which of the following diagnoses? a. Leiomyoma b. Pregnancy c. Ovarian cancer d. All of the above 22. A 48-year-old woman is seen in the clinic with complaints of prolonged heavy menstrual periods. She is pale and states she can no longer exercise. Pelvic exam reveals a single, very large mass. Which of the following diagnostic tests should the clinician order first? a. Transvaginal ultrasound b. Endometrial biopsy c. MRI d. Abdominal computed tomography scan 23. A 45-year-old woman is seen because of irregular menstrual periods. Her follicle-stimulating hormone (FSH) level is 48 mIU/mL, and her luteinizing hormone (LH) level is elevated. She asks the clinician what this means. Which would be the best response? a. “You are approaching menopause.” b. “You have a hormonal imbalance.” c. “Your FSH is normal, but your pituitary is making too much LH.” d. “There is an imbalance between your ovaries and pituitary.” 24. Which of the following tests is essential for a 46-year-old woman who the clinician suspects is perimenopausal? a. Pregnancy b. Estrogen level c. Progesterone level d. LH level 25. A 60-year-old woman is seen for an annual checkup. Her obstetric history reveals para 6, gravida 6. She reports that she went through menopause at age 45. Her grandmother died at age 80 of colon cancer, and her father died of lung cancer. What in her history would be a risk factor for ovarian cancer? a. Her numerous pregnancies b. Her age at menopause c. Her father’s history of lung cancer d. Her grandmother’s history of colon cancer 26. A 58-year-old woman, who had a total abdominal hysterectomy at the age of 45, is diagnosed with atrophic vaginitis. Which of the following is the most appropriate treatment? a. Conjugated estrogen, 0.625 mg/day oral b. Estradiol, 7.5 mcg/24 hr vaginal ring c. Medroxyprogesterone, 10 mg/day oral d. Conjugated estrogen, 0.3 mg + medroxyprogesterone 1.5 mg/day oral True/False Indicate whether the statement is true or false. 1. Oral contraceptive pills can cause endometrial cancer. 2. A woman taking estradiol is at risk for developing endometrial cancer. 3. Most breast cancer cases are in women with a family history of breast cancer. Sexually Transmitted Infections Multiple Choice Identify the choice that best completes the statement or answers the question. 1. During data collection the nurse notes the presence of a chancre on a male patients penis. For which sexually transmitted infection should the nurse focus additional data collection? a. Herpes b. Syphilis c. Gonorrhea d. Chlamydia 2. A patient is diagnosed with a parasitic infection caused by close contact with another persons genitals. For which infection should the nurse plan care? a. Phthirus pubis b. Treponema pallidum c. Neisseria gonorrhoeae d. Chlamydia trachomatis 3. It is documented in the medical record that a patient has gummas. For which sexually transmitted infection should the nurse plan care? a. Syphilis b. Gonorrhea c. Chlamydia d. Genital herpes 4. The nurse is assisting with teaching a 22-year-old female patient who is diagnosed with a sexually transmitted infection (STI). She says, I dont understand. My boyfriend always wears a condom. Which understanding by the nurse should guide teaching in this situation? a. Condoms are a reliable source of protection against STIs. b. It is a myth that condoms provide any protection against STIs. c. Condoms can decrease the risk of STIs, but they are not foolproof. d. Condoms must be used with a spermicide to guarantee protection against STIs. 5. The nurse is providing care for a patient with genital herpes who has vesicular lesions. What term should the nurse use to describe these lesions to the patient? a. Warts b. Rashes c. Blisters d. Papules 6. Human papillomavirus (HPV) produces verrucous growths. What term should the nurse use to describe these lesions to the patient? a. Warts b. Rashes c. Blisters d. Papules 7. The nurse is collecting data on a patient with Chlamydia. Which assessment finding should be reported immediately to the RN or physician? a. Painful urination b. Red conjunctivae c. Vaginal discharge d. Sharp pain at the base of the ribs 8. Because Trichomonas is relatively large, unusually shaped, and diagnosed quickly, the nurse is asked to assist the physician obtain which type of specimen? a. Culture b. Blood test c. Wet mount d. Litmus paper 9. A patient diagnosed with Trichomonas asks the nurse how the diagnosis will affect her risk for cervical cancer. Which response by the nurse is best? a. Wet-mount slides should be done yearly to help detect cervical cancer. b. Serological testing will be done to detect tumor proteins and screen for cervical cancer. c. Papanicolaou smears should be done more frequently because results may be altered by Trichomonas. d. Culture and sensitivity testing is done with Papanicolaou (Pap) smears every other year to determine if you have cervical cancer. 10. A patient asks why the physician has recommended systemic interferon treatment for genital warts. Which explanation should the nurse provide to the patient? a. Interferon can improve liver function. b. Interferons can increase your red blood cell count. c. Interferon treatment does not have any side effects. d. Interferon therapy can attack warts all over the body at the same time. 11. A patient with hepatitis B virus (HBV) delivers a 6-pound 2-ounce baby boy. Which action should the nurse anticipate will be needed for the infant? a. Intravenous antibiotics for 12 hours b. Antiviral eye medication less than 2 hours after birth c. There is no treatment that is safe and effective for infants. d. HBV-immune globulin less than 12 hours after birth and then HBV vaccine series 12. The nurse must bathe a patient with herpes. What is the nurses best protection against contracting sexually transmitted infections (STIs) from patients while providing perineal hygiene? a. Wearing gloves at all times b. Washing hands following care c. Practicing standard precautions d. Avoiding touching patients who have STIs 13. The nurse is caring for a pregnant woman who is fearful that her unborn child will be born blind because of having a sexually transmitted infection (STI). For which STI should the nurse plan care to prevent ophthalmia neonatorum in the newborn? a. Syphilis b. Gonorrhea c. Genital warts d. Genital herpes 14. The nurse is caring for a young woman who is newly diagnosed with genital warts. She states, I heard you can get cancer from STIs. Is that true? Which response by the nurse is correct? a. No, you cannot get cancer from STIs. b. Yes, most STIs can lead to cancerous changes if not treated promptly. c. Yes, some STIs have been linked to cancer, so adequate treatment is very important. d. No, that is not true, but a diagnosis of cancer does increase the risk of contracting an STI. 15. The nurse is identifying ways for a young adult to reduce the risk of contracting a sexually transmitted infection (STI). What should the nurse teach about the relationship between consumption of alcohol and immediate risk of contracting an STI? a. Alcohol may reduce inhibitions. b. Alcohol increases risk for liver disease. c. Alcohol lowers the bodys resistance to infection. d. Alcohol impairs the integrity of the mucous membranes, providing a portal of entry for infection. 16. The nurse reviews the ways to prevent condom breakage with a patient. Which patient statement indicates that more teaching is necessary? a. Condoms should never be reused. b. I should use a water-soluble lubricant. c. Before I use a condom, I should inflate it and check it for holes and leaks. d. I should make sure to leave a half inch extra space at the end of the condom. 17. The nurse is assisting with the admission of a known intravenous drug abuser to a medical unit. In addition to drug abuse, which disorder in the patients history is most consistent with a diagnosis of hepatitis? a. Jaundice b. Diabetes mellitus c. Bowel obstruction d. Chronic headaches 18. The nurse is teaching a patient the importance of completing treatment for gonorrhea. On which information is the nurse basing this teaching? a. Gonorrhea is not treatable. b. Only men experience symptoms; women are usually asymptomatic. c. Men and women may be asymptomatic and still transmit the infection. d. Treatment is associated with many serious side effects, so compliance is low. 19. The nurse is assisting in the preparation of a teaching seminar for adolescents to prevent the development of a sexually transmitted infection (STI). Which nonsexual activity should the nurse teach that may transmit a sexually transmitted infection (STI)? a. Sharing a cigarette b. Borrowing a hairbrush c. Coughing and sneezing d. Sharing intravenous drug equipment 20. A patient asks for the best way to prevent contracting a sexually transmitted infection (STI). What response should the nurse make to this patients question? a. Abstinence b. Oral contraceptives c. Condom with spermicide d. Prophylactic oral antibiotics 21. A patient diagnosed with genital warts asks how they developed. Which pathogen should the nurse explain as causing genital warts? a. Sarcoptes scabiei b. Hepatitis A and B c. Human papillomavirus d. Chlamydia trachomatis 22. The nurse is caring for a 76-year-old retired man who is undergoing evaluation for dementia. What would be an important part of the mans history to report to the physician? a. The patient has a history of syphilis. b. The patient was exposed to Chlamydia. c. The patient has a history of hepatitis B. d. The patient has a history of genital warts. 23. A patient is undergoing treatment that involves the burning of lesions with heat or chemical agents. The nurse recognizes that this patient most likely has which condition? a. Syphilis b. Chlamydia c. Hepatitis B d. Genital warts 24. The nurse is providing care for a newborn. Which intervention should the nurse make to prevent development of ophthalmia neonatorum? a. Interferon injection b. Antibiotic eyedrops c. Vitamin K injection d. Hepatitis B virus (HBV)-immune globulin 25. While reviewing a medical record the nurse notes that patient has a strawberry cervix. For which sexually transmitted infection (STI) would the nurse plan care? a. Gonorrhea b. Herpes simplex c. Trichomoniasis d. Human papillomavirus infection 26. The nurse is preparing a poster presentation identifying the frequency of sexually transmitted infections (STIs) in the United States. Which STI should the nurse highlight as being the most commonly diagnosed? a. Gonorrhea b. Chlamydia c. Trichomoniasis d. Human papillomavirus 27. While assisting a health care provider (HCP) conduct a pelvic examination, the patient complains of severe pain during the bimanual examination. For which health problem should the nurse suspect this patient is going to need care? a. Syphilis b. Gonorrhea c. Pelvic inflammatory disease d. Human papillomavirus infection 28. While assisting with care, the nurse counsels the patient diagnosed with a sexually transmitted infection (STI) about notification of sexual partners. Which patient statement indicates the need for further teaching? (Select all that apply.) a. I can contact my sexual partners myself. b. Reporting regulations are the same throughout the country. c. A report form will be completed in my chart that includes a list of my sexual contacts. d. The public health authority can notify a list of sexual contacts without including my identity. Multiple Response Identify one or more choices that best complete the statement or answer the question. 29. The nurse is assisting with teaching a patient who has been exposed to hepatitis B. Which symptoms should the nurse explain may occur before jaundice appears? (Select all that apply.) a. Rash b. Nausea c. Confusion d. Dark-colored urine e. Muscle or joint pain f. Elevated blood glucose 30. The nurse is reviewing prescribed laboratory tests for a patient demonstrating manifestations of syphilis. What diagnostic tests should the nurse expect to be prescribed for this patient? (Select all that apply.) a. RPR b. NAT c. VDRL d. ELISA e. Culture f. CD4 counts 31. A 24-year-old woman diagnosed with Chlamydia has been prescribed doxycycline. What should be included in the nurses teaching about the drug treatment? (Select all that apply.) a. Take this drug with a meal. b. Do not take with dairy products. c. Avoid unnecessary exposure to sunlight. d. Abstain from alcohol for at least 48 hours after treatment. e. Use birth control methods to ensure you do not become pregnant. 32. The nurse is teaching a patient about the use of condoms to prevent sexually transmitted infections (STIs). Which information should the nurse include in this teaching? Select all that apply. a. Condoms can decrease the risk of transmitting STDs. b. Latex condoms are less likely to break than other types. c. Inflating the condom prior to use allows for effective inspection. d. Condoms should be used no more than twice and then discarded properly. e. Use of a water-soluble lubricant with a condom increases its effectiveness in preventing the spread of an STD. f. Use of a petroleum-based lubricant with a condom increases its effectiveness in preventing the spread of an STD. 33. The nurse is providing care for a patient recently diagnosed with Chlamydia. Which information should the nurse recommend be included in patient teaching? (Select all that apply.) a. Women with Chlamydia may complain of a sore throat. b. Chlamydia is characterized by the development of chancres. c. Ophthalmia neonatorum is seen in infants born to women with Chlamydia. d. Chlamydia can be transmitted sexually and by blood and body fluid contact. e. The risk of ectopic pregnancy is increased in women with a history of Chlamydia. f. The Chlamydia virus can lie dormant in the nervous system tissues and reactivate when an individual is under stress or has a compromised immune system. 34. The nurse notes that a patient is diagnosed with vulvovaginitis. What should the nurse expect when assessing this patient? (Select all that apply.) a. Vaginal edema b. Vaginal discharge c. Areas of ecchymosis d. Dark brown vaginal bleeding e. Complaints of vaginal itching and burning 35. A patient in labor is diagnosed with mucopurulent cervicitis. For which health problems should the nurse anticipate providing care to the newborn? (Select all that apply.) a. Pneumonia b. Conjunctivitis c. Irregular heart rate d. Flaccid extremities e. Cyanotic extremities 36. A patient diagnosed with syphilis reminds the HCP of having an allergy to penicillin. Which medications should the nurse expect to be prescribed for this patient? (Select all that apply.) a. Gentamicin b. Amoxicillin c. Tetracycline d. Doxycycline e. Erythromycin 37. While providing a bath the nurse suspects that an older female patient has a Trichomonas infection. What type of discharge did the nurse observe to come to this conclusion? (Select all that apply.) a. Frothy discharge b. Foul-smelling discharge c. Yellow-green discharge d. Open sores on the labia majora e. Wart-like growths on the labia minora Common Musculoskeletal Complaints Multiple Choice Identify the choice that best completes the statement or answers the question. 1. One of the initial steps in assessing patients with musculoskeletal complaints is to determine whether the complaint is articular or nonarticular in origin. Which of the following is an example of an articular structure? a. Bone b. Synovium c. Tendons d. Fascia 2. You have detected the presence of crepitus on examination of a patient with a musculoskeletal complaint. Additionally, there is limited range of motion (ROM) with both active and passive movement. These findings suggest that the origin of the musculoskeletal complaint is: a. Articular b. Inflammatory c. Nonarticular d. A and B 3. Which of the following signs or symptoms indicate an inflammatory etiology to musculoskeletal pain? a. Decreased C-reactive protein b. Hyperalbuminemia c. Morning stiffness d. Weight gain 4. Which of the following statements concerning the musculoskeletal examination is true? a. The uninvolved side should be examined initially and then compared to the involved side. b. The part of the body that is causing the patient pain should be examined first. c. When possible, the patient should not be asked to perform active range-of-motion (ROM) exercises to avoid causing pain. d. Radiographs should always be obtained prior to examination so as not to cause further injury to the patient. 5. You are performing muscle strength testing on a patient presenting with musculoskeletal pain and find that the patient has complete ROM with gravity eliminated. Which numeric grade of muscle strength would you give this patient? a. 1 b. 2 c. 3 d. 4 e. 5 6. Mrs. Gray is a 55-year-old woman who presents with tightness, pain, and limited movement in her right shoulder. She denies any history of trauma. Her examination reveals a 75% reduction in both active and passive ROM of the right shoulder. Mrs. Gray also is experiencing tenderness with motion and pain at the deltoid insertion. Her medical history is significant for type 1 diabetes mellitus and hypertension. Her social history reveals that she is a secretary and that she is right- handed. Based on her examination and medical history, you suspect adhesive capsulitis, or “frozen shoulder.” Which clue in Mrs. Gray’s history supports this diagnosis? a. History of hypertension b. Her affected shoulder is also her dominant arm. c. Her history of diabetes mellitus d. Her work as a secretary predisposes her to repetitive motions. 7. Jennifer is an 18-year-old who comes to the emergency room after a fall during a soccer game. Jennifer explains that she fell on her left side and kept her arm out straight to break her fall. She has been experiencing severe pain and limited ROM in her left shoulder. The clinician has diagnosed Jennifer with a dislocated shoulder. Which of the following statements are true concerning shoulder dislocation? a. Posterior dislocations are more common than anterior dislocations. b. There is a risk of neurovascular and neurosensory trauma, so the clinician should check for distal pulses. c. Recurrent dislocations are uncommon and would require great force to result in injury. d. Surgery is most commonly the treatment of choice. 8. Mrs. Anderson is a 35-year-old woman who has been recently diagnosed with carpal tunnel syndrome. She has two young children and asks the clinician what the chances are that they also will develop carpal tunnel syndrome. Which of the following responses would be correct regarding the risk of developing carpal tunnel syndrome? a. Carpal tunnel syndrome commonly occurs in families. Genetic factors are thought to account for about one-half the risk of developing carpal tunnel. b. Only people with occupations that require repeated flexion extension of the wrist, use of hand tools that require forceful gripping, or use of hand tools that vibrate are at risk for developing carpal tunnel. c. An underlying musculoskeletal disorder must be present for a person to develop carpal tunnel. d. Carpal tunnel syndrome only occurs in the presence of a hormonal imbalance. 9. Which of the following statements is true regarding the treatment of carpal tunnel syndrome? a. The goal of treatment is to prevent flexion and extension movements of the wrist. b. Splints are used in carpal tunnel syndrome, because they allow for free movement of the fingers and thumb while maintaining the wrist in a neutral position. c. Corticosteroid injections are discouraged in the treatment of carpal tunnel syndrome because of the risks for median nerve damage, scarring, and infection. d. All of the above 10. Sam is a 25-year-old who has been diagnosed with low back strain based on his history of localized low back pain and muscle spasm along with a normal neurological examination. As the clinician, you explain to Sam that low back pain is a diagnosis of exclusion. Which of the following symptoms would alert the clinician to the more serious finding of a herniated nucleus pulposus or ruptured disc? a. Morning stiffness and limited mobility of the lumbar spine b. Unilateral radicular pain symptoms that extend below the knee and are equal to or greater than the back pain c. Fever, chills, and elevated erythrocyte sedimentation rate d. Pathologic fractures, severe night pain, weight loss, and fatigue 11. The clinician has instructed Sam, a 25-year-old patient with low back strain, to use NSAIDs to manage his symptoms of pain and discomfort. Which of the following statements would be most appropriate when teaching Sam about the use of NSAIDs? a. “You should start with the lowest dose that is effective in managing your pain, because long-term use of NSAIDs can result in gastrointestinal (GI) disorders such as ulcers and hemorrhage.” b. “You should start with the lowest dose that is effective in managing your pain to avoid developing tolerance to the medication.” c. “You should take the maximum recommended dose of NSAIDs so that you will not need to take narcotics to control your pain.” d. “It is important to take NSAIDs on an empty stomach in order to increase absorption.” 12. Janet is a 30-year-old who has recently been diagnosed with a herniated disc at the level of L5-S1. She is currently in the emergency room with suspicion of cauda equina compression. Which of the following is a sign or symptom of cauda equina compression? a. Gastrocnemius weakness b. A reduced or absent ankle reflex c. Numbness in the lateral foot d. Paresthesia of the perineum and buttocks 13. Which of the following statements is true concerning the management of the client with a herniated disc? a. Muscle relaxants and narcotics can be used to control moderate pain but should be discontinued after 3 weeks of use. b. An epidural injection is helpful in reducing leg pain that has persisted for at least 3 weeks after the herniation occurred. c. Intolerable pain for more than a 3-month period is an indication for surgical intervention. d. All of the above 14. John is a 16-year-old boy who presents to the emergency room after hurting his knee in a football game. He described twisting his knee and then being unable to extend it completely. John tells the clinician that he heard a pop when the injury occurred and has been experiencing localized pain. The clinician suspects a meniscal tear. Which test would be most appropriate to assess for the presence of a meniscal tear? a. Valgus stress test b. McMurray circumduction test c. Lachman test d. Varus stress test 15. The clinician suspects that a client has patellar instability. In order to test for this, the client is seated with the quadriceps relaxed, and the knee is placed in extension. Next the patella is displaced laterally, and the knee flexed to 30°. If instability is present, this maneuver displaces the patella to an abnormal position on the lateral femoral condyle, and the client will perceive pain. Testing for patellar instability in this way is known as: a. Apprehension sign b. Bulge sign c. Thumb sign d. None of the above 16. The clinician is caring for Diane, a 22-year-old woman who presents with an injured ankle. Diane asks the clinician if she will need an x-ray. The clinician explains to Diane that an x-ray is not always necessary for an injured ankle and that the decision to obtain radiographs is dependent on the examination and Diane’s description of her injury. Which of the following clues in Diane’s examination or history would alert the clinician to the need for obtaining radiographs? a. Inability to bear weight immediately after the injury b. Development of marked ankle swelling and discoloration after the injury c. Crepitation with palpation or movement of the ankle d. All of the above 17. Mr. Jackson is a 65-year-old man recently diagnosed with osteoarthritis. The clinician has explained to Mr. Jackson that the goals for managing osteoarthritis include controlling pain, maximizing functional independence and mobility, minimizing disability, and preserving quality of life. Mr. Jackson explains to the clinician that his first choice would be to use complementary therapies to control his condition and asks what therapies are most effective in treating osteoarthritis. What would be the most appropriate response from the clinician? a. “Complementary therapies should be considered only if surgical interventions are not successful.” b. “I am unfamiliar with the available complementary therapies for osteoarthritis and prefer to discuss more mainstream treatments, such as NSAIDs and physical therapy, to manage your condition.” c. “I would be happy to discuss all the treatment options available to you. Complementary therapies, such as acupuncture, acupressure, and tai-chi, are being studied for use in the treatment of osteoarthritis and have shown promise when used with standard medical therapy.” d. “It would be crazy to use complementary therapies to treat such a serious condition.” 18. Normal estrogen function is important for preventing osteoporosis in both men and women. Estrogen works to prevent osteoporosis in which of the following ways? a. By decreasing the erosive activity of osteoclasts b. By promoting osteoclastogenesis c. By inhibiting osteoclast apoptosis d. All of the above 19. Which of the following tests is considered the gold standard for definitively diagnosing osteoporosis? a. Bone alkaline phosphatase levels b. Urinary N-telopeptide assay c. Bone mass density measurement by densitometry d. Magnetic resonance imaging 20. What is the recommended daily calcium intake for adults over the age of 50 with low bone mass? a. 1,200 mg/day b. 1,000 mg/day c. 1,300 mg/day d. 1,500 mg/day 21. Mrs. Allen is a 60-year-old woman who has been diagnosed with osteoporosis. She is very concerned about the risk of breast cancer associated with hormone replacement therapy and is wondering what other treatments are available to her. The clinician explains that bisphosphonates are another class of drugs used in the prevention and treatment of osteoporosis. What teaching should the clinician give Mrs. Allen in regard to taking bisphosphonates? a. Taking bisphosphonates can result in hypercalcemia, so calcium intake should be decreased while taking this class of drugs. b. There is potential for upper GI irritation, so these medications are contraindicated in people with abnormalities of the esophagus or delayed esophageal emptying. c. This class of drugs can be taken at any time of the day without regard to meals. d. None of the above 22. Which stage of Paget’s disease is characterized by elevated numbers of osteoblasts, resulting in abnormal increases in bone remodeling and leading to an irregular deposition of collagen fibers? a. Lytic b. Mixed c. Sclerotic d. All of the above 23. Which of the following statements concerning the treatment of fibromyalgia syndrome is true? a. There is currently no cure for the disorder; however, patients should be made aware that symptom relief is possible. b. Treatment is directed toward controlling discomfort, improving sleep, and maintaining function. c. Fibromyalgia syndrome can be difficult to manage, requiring a variety of approaches and multiple medications. d. All of the above 24. One of the most frequent presenting signs/symptoms of osteoporosis is: a. Goiter b. Abnormal serum calcium c. Elevated urine biochemical markers d. Bony fracture 25. Mrs. Thomas was seen in the office complaining of pain and point tenderness in the area of her elbow. The pain has increased following a day of gardening one week ago. A physical finding that differentiates the diagnosis and is most consistent with lateral epicondylitis (tennis elbow) is: a. Ecchymosis, edema, and erythema over the lateral epicondyle b. Pain at the elbow with resisted movements at the wrist and forearm c. Inability to supinate and pronate the arm d. Inability to flex or extend the elbow against resistance 26. A 70-year-old female has fallen 2 weeks ago and developed immediate pain in her left wrist. She thought she just bruised it but is worried because it has not improved. She has used Tylenol® and ice at home, and that has helped slightly. You examine her and find she has moderate swelling and ecchymosis but no overtly obvious deformity. Her ROM is uncomfortable and severely diminished due to the pain. No crepitus is heard or felt. Her fingers are warm; her pulse is strong; and capillary refill is less than 2 seconds. What should you do? a. Make an immediate referral for an orthopedic evaluation without further assessment. b. Tell her that it takes time for these bruises to improve, so she should be patient. c. Obtain a wrist x-ray and place her wrist in a splint or prescribe a splint. d. Send her to the emergency room for reduction of this obvious wrist fracture. True/False Indicate whether the statement is true or false. 1. Osteoarthritis is primarily a noninflammatory condition. 2. The presence of a positive rheumatoid factor is always indicative of rheumatoid arthritis. Spinal Disorders 1. The nurse is caring for a home health patient who had a spinal cord injury at C5 three years ago. The nurse bases the plan of care on the knowledge that the patient will be able to: a. feed self with setup and adaptive equipment. b. transfer self to wheelchair. c. stand erect with full leg braces. d. sit with good balance. ANS: A A cord injury at C5 allows for ability to drive an electric wheelchair with mobile hand supports and feed self with adaptive equipment. 2.A frantic family member is distressed about the flaccid paralysis of her son following a spinal cord injury several hours ago. What does the nurse know about this condition? a. It is an ominous indicator of permanent paralysis. b. It is possibly a temporary condition and will clear. c. It degenerates into a spastic paralysis. d. It will progress up the cord to cause seizures. ANS: B A period of flaccid paralysis following a cord injury is called areflexia, or spinal shock, and may be temporary. 3.A patient with a spinal cord injury at T1 complains of stuffiness of the nose and a headache. The nurse notes a flushing of the neck and goose flesh. What should be the primary nursing intervention based on these assessments? a. Place patient in flat position and check temperature b. Administer oxygen and check oxygen saturation c. Place on side and check for leg swelling d. Sit upright and check blood pressure ANS: D These are indicators of autonomic dysreflexia or hyperreflexia. It is a medical emergency. The patient should be placed in an upright position to decrease blood pressure and the blood pressure should be checked. Assessments for impaction, full bladder, or a urine infection can help to evaluate this condition. 4. The nurse is aware that the characteristic gait of the person with Parkinson disease is a propulsive gait, which causes the patient to: a. stagger and need support of a walker. b. shuffle with arms flexed. c. fall over to one wide when walking. d. take small steps balanced on the toes. ANS: B The propulsive gait causes the patient to shuffle with his arms flexed and with a loss of postural reflexes. 5. The newly admitted patient to the emergency room after a motorcycle accident has serosanguineous drainage coming from the nose. What is the most appropriate nursing response to this assessment? a. Cleanse nose with a soft cotton-tipped swab b. Gently suction the nasal cavity c. Gently wipe nose with absorbent gauze d. Ask patient to blow his nose ANS: C The patients ear and nose are checked carefully for signs of blood and serous drainage, which indicate that the meninges are torn and spinal fluid is escaping. No attempt should be made to clean out the orifice or to blow the nose. The drainage can be wiped away. The drainage can be tested for the presence of glucose, which would confirm that the fluid is spinal fluid and not mucus. 6. How would the nurse instruct a patient with Parkinson disease to improve activity level? a. To use a soft mattress to relax the spine b. To walk with a shuffling gait to avoid tripping c. To walk with hands clasped behind back to help balance d. To sit in hard chair with arms for posture control ANS: C The patient with Parkinson disease can improve the activity level by sleeping on a firm mattress without a pillow to prevent spinal curvature, hold hands clasped behind to keep better balance, and keep the arms from hanging stiffly at the side. Walk with a lifting of the feet to avoid tripping and freezing. Soft-Tissue Disorders _ 1. The nurse is contributing to the plan of care for a patient who has a right fractured femur. What intervention should the nurse include in the plan of care to prevent fat emboli? a. Decrease dietary consumption of fats. b. Maintain immobilization of the right leg. c. Encourage coughing and deep breathing hourly. d. Perform passive range of motion on the right leg. 2. A patient has an open reduction of a radial fracture and is casted. Several hours after the operation, the patient reports a throbbing pain in the arm. What nursing action is essential for the nurse to take? a. Reposition arm. b. Perform neurovascular checks. c. Administer analgesics as ordered. d. Notify the physician immediately. 3. The nurse is monitoring a patient with a casted left tibial fracture and a contusion of the thigh. The patient reports increasing pain in the left foot that has not been relieved by morphine injections. What should the nurse do? a. Reposition the casted leg. b. Repeat the morphine injection now. c. Give a higher ordered dose of morphine. d. Ensure physician is immediately notified. 4. The nurse finds a 2-day postoperative patient who had a right total hip replacement lying supine with crossed legs. What data should the nurse collect on this patient? a. The right leg for shortening b. The right knee for crepitation c. The left leg for internal rotation d. The left leg for loss of function 5. The nurse is caring for a patient who had a closed reduction of the ulna with a cast applied. Later the patient reports left arm pain. What should the nurse do first? a. Pad the edges of the cast. b. Notify the physician immediately. c. Administer an analgesic as ordered. d. Perform neurovascular check on fingers. 6. The nurse is reinforcing teaching provided to a patient recovering from right total hip replacement. Which patient statement indicates a correct understanding of the teaching? a. Keep legs apart. b. Lie prone in bed. c. Move right leg closer to the left leg. d. Do not bear any weight on the left leg. 7. A patient with a casted, fractured left leg asks why the leg has to be elevated. What should the nurse respond to this patient? a. Decreases swelling. b. Prevents cast cracking. c. Increases your comfort. d. Allows the cast to dry evenly. 8. The nurse is caring for a patient who has had a right hip replacement. For which position is the nurse attempting to achieve when a pillow is placed between the legs during turning? a. Flexion of the knees b. Abduction of the thighs c. Adduction of the hip joint d. Hyperextension of the knees 9. The nurse sees a neighbor fall and fracture a leg. What should the nurse do first for the neighbor? a. Assess pain. b. Transport to an emergency department. c. Cover site of open fracture with clean dressing. d. Immobilize the affected limb using minimal movement. 10. The nurse is reinforcing teaching provided to a patient with rheumatoid arthritis (RA). Which patient statement indicates understanding of the symptoms of RA? a. Fatigue b. Paralysis c. Crepitation d. Shortness of breath 11. A patient with a 36-hour-old fractured femur is in traction and is prescribed morphine 10 mg every 3 hours as needed. The patient received a dose 3 hours ago and is now reporting a pain level of 8. The patient is stable. Which action should the nurse take? a. Hold medication. b. Notify the registered nurse (RN). c. Give pain medication as ordered. d. Give pain medication in 30 minutes. 12. The nurse is caring for a patient who has a newly casted, fractured wrist. Data collection reveals slightly puffy fingers with good capillary refill. What should the nurse do now to prevent complications? a. Notify the RN. b. Apply heat to the cast. c. Elevate the cast on pillows. d. Remove the pillow under the cast. 13. A patient with gout has been instructed on the prescribed medication allopurinol (Zyloprim). Which patient statement indicates understanding of the action of this medication? a. Excretes proteins. b. Blocks formation of uric acid. c. Increases formation of purines. d. Increases metabolism of purines. 14. The nurse is evaluating teaching provided to a patient with gout. Which patient menu selection indicates that additional teaching is required? a. Pike b. Bass c. Perch d. Sardines 15. The nurse is reinforcing teaching provided to a patient with gout. Which food should the patient state will be avoided that indicates teaching has been effective? a. Rice b. Beets c. Liver d. Bananas 16. The nurse is contributing to the plan of care for a patient with Pagets disease. Which outcome should the nurse identify as being appropriate for this patient? a. Gain 5 lb weekly. b. Intake equals output. c. Identify coping skills. d. Pain is relieved at a satisfactory level. 17. The nurse is contributing to the plan of care for a patient who has a fractured hip and is placed in Bucks (boot) traction while awaiting surgery. What is the desired outcome for placing the patient in Bucks traction? a. Restrain patient. b. Realign fracture. c. Relieve patient pain. d. Maintain fracture reduction. 18. The nurse is reinforcing teaching for a patient who has had a total hip replacement on correct sitting positions. Which position should the nurse teach the patient to avoid? a. Crossing legs b. Elevating legs c. Flexing ankles d. Extending knees 19. The nurse is contributing to the plan of care for a patient who has an upper extremity amputation. Why should the nurse keep in mind that this type of amputation can be more debilitating than a lower extremity amputation when planning care? a. The upper extremity is more visible. b. Prosthetic fitting is easier for the leg. c. The upper extremity is more specialized. d. There is greater blood supply to the upper extremity. 20. The nurse observes a petechial rash and respiratory distress in a patient recovering from a fractured femur. What should these findings suggest to the nurse? a. Infection b. Pneumonia c. Fat embolism d. Pleural effusion 21. A patient who has a displaced mid-shaft fracture of the left femur and is in balanced suspension skeletal traction with 35 pounds of weights is experiencing calf pain with right foot dorsiflexion. Which action should the nurse take? a. Notify the RN. b. Check the traction setup. c. Reduce 5 pounds of weight. d. Encourage dorsiflexion more frequently. 22. The nurse is contributing to the plan of care for a patient who is scheduled for a below-the- knee amputation. What nursing diagnosis should be recommended for the preoperative plan of care? a. Anxiety b. Self-Care Deficit c. Fluid Volume Deficit d. Ineffective Airway Clearance 23. The nurse is reinforcing teaching on positioning for a patient after a right total knee replacement. Which patient statement indicates a correct understanding of the teaching? a. Prone. b. Side lying. c. Supine with pillow under right knee. d. Supine with three pillows between legs. 24. The nurse is reinforcing teaching provided to a patient for carpal tunnel syndrome treatment. Which patient statement indicates a correct understanding of the teaching? a. Bedrest. b. Arm sling. c. Wrist splint. d. Hand exercises. 25. A patient with a fractured pelvis and a left acetabular fracture is prescribed bedrest. When the patient asks to toilet, which measure would be appropriate? a. Help patient up on a commode very carefully. b. Turn patient onto right side, place the bedpan behind, and turn back. c. Have patient sit up as high as possible and lift self up with hands pushing on the bed, then slide the bedpan underneath. d. Ask patient to lift straight up using a trapeze mounted above the bed and slide a bedpan underneath from the right side. 26. The nurse is caring for a patient with gout. Which laboratory value should the nurse review which indicates that the treatment plan is effective? a. Uric acid: 7.9 mg/dL b. Creatinine: 0.8 mg/dL c. Blood urea nitrogen: 15 mg/dL d. Low-density lipoprotein (LDL): 115 mg/dL 27. The nurse is reinforcing teaching provided to a patient who is postmenopausal, has lost 2 inches of height, and has osteoporosis. Which patient statement indicates correct understanding of the purpose of calcium supplements? a. To decrease bone loss b. To increase energy levels c. To decrease serum calcium d. To increase excretion of calcium 28. A patient is completing instructions about complications that can occur from osteoporosis. Which complication should the patient state as evidence that teaching has been effective? a. Hip fracture. b. Overgrowth of bone. c. Bone spur formation. d. Increased bone density. 29. The nurse is reviewing data collected during the health history for a patient with osteoporosis. What should the nurse identify as a risk factor for osteoporosis development? a. Daily use of antacid b. Walking 1 mile daily c. Increased caffeine intake d. Increased dairy food intake 30. The nurse reinforces medication teaching provided to a patient with rheumatoid arthritis. Which medication should the patient identify as helpful to control the symptoms of the health problem? a. Digoxin. b. Ibuprofen. c. Morphine. d. Penicillin. 31. The nurse checks a patients casted right leg resting upon a pillow and finds that the cast appears too tight. What should the nurse do? a. Notify the RN. b. Administer pain medication. c. Apply an extra blanket to the leg. d. Remove the pillow under the cast. 32. The nurse is contributing to the plan of care for a patient who has a bone fracture that is splintered and has shattered into numerous fragments. Which term should the nurse use to document this type of fracture? a. Impacted b. Avulsion c. Greenstick d. Comminuted 33. The nurse reinforces teaching on prevention of osteomyelitis with a patient who has an open fracture of the right leg. Which patient statement indicates that teaching has been effective? a. Apply ice to right leg. b. Keep leg immobilized. c. Increase calcium intake in diet. d. Wash hands prior to touching fracture area. 34. An 87-year-old female with a history of osteoarthritis reports an average generalized pain score of 4 on a 0-to-10 scale while using acetaminophen prn. Which response about this pain level should the nurse make to the patient? a. Do you take a daily calcium supplement? b. Im glad the acetaminophen is working for you. c. Are you satisfied with this level of pain control? d. Research shows that acetaminophen is not really effective for osteoarthritis pain. 35. A patient is diagnosed with osteomyelitis of the right lower leg. What should the nurse expect to be prescribed for this patients care? a. Anticoagulant therapy b. Casting of the extremity c. Fasciotomy of the wound d. Long-term antibiotic therapy Multiple Response Identify one or more choices that best complete the statement or answer the question. 36. A patient 48 hours after surgery for a fractured femoral shaft is experiencing mental confusion, tachycardia, tachypnea, and dyspnea. The patients blood pressure is elevated and petechiae are present on the chest. After reporting the findings to the RN what should the nurse do while awaiting the physicians specific orders? (Select all that apply.) a. Administer oxygen. b. Prepare patient for arterial blood gas tests. c. Prepare patient for chest x-ray or lung scan. d. Maintain bedrest and keep movement to a minimum. e. Ask patient to move affected limb to see if pain is worse. f. Place patient in high Fowlers position or raise the head of the bed. 37. A patient asks the difference between osteoarthritis and rheumatoid arthritis. What manifestations should the nurse explain are characteristic of rheumatoid arthritis? (Select all that apply.) a. Low-grade fever b. Heberdens nodes c. Autoimmune disease d. Activity increases pain e. Early morning stiffness f. Involvement of other major organs 38. The nurse is collecting data from a patient suspected of developing a fat embolus from a fracture of the right femur. Which manifestations should the nurse expect? (Select all that apply.) a. Petechiae b. A migraine c. Tachycardia d. Mental confusion e. Numbness in the right leg f. Muscle spasms in the right thigh 39. The nurse is caring for a patient in traction. Which actions are appropriate when caring for this patient? (Select all that apply.) a. Allow weights to hang freely in place. b. Use assistance to reposition the patient in bed. c. Hold weights up if the patient is shifting position in bed. d. Remove weights if the patient is being moved up in bed. e. Lighten weights for short periods if the patient reports pain. 40. The nurse is contributing to the plan of care for a patient recovering from total hip replacement. Which exercises should the nurse recommend to help prevent deep vein thrombosis (DVT) formation? (Select all that apply.) a. Foot circles b. Toe touches c. Heel pumping d. Deep knee bends e. Quadriceps setting f. Straight leg raises (SLRs) 41. A patient in the ambulatory clinic is diagnosed with a muscle strain. What actions should the nurse instruct the patient to do to treat this injury? (Select all that apply.) a. Rest the limb. b. Elevate the limb. c. Apply heat for 1 hour. d. Apply ice to the area. e. Wrap with an elastic bandage. 42. The nurse is caring for a patient with a minor rotator cuff shoulder injury. What should the nurse emphasize when reviewing care with this patient? (Select all that apply.) a. Apply ice b. Rest the shoulder c. Take NSAIDs as prescribed d. Begin out-patient physical therapy e. Use 2 lb hand weights for exercising 43. During a health history the nurse becomes concerned that a patient is at risk for developing osteoporosis. Which modifiable risk factors did the nurse use to come to this conclusion? (Select all that apply.) a. Small boned b. Postmenopausal c. Cigarette smoking d. Sedentary lifestyle e. Low calcium intake 44. The nurse is assisting in the development of an educational seminar on prevention of osteoporosis for a group of community members. Which actions should the nurse suggest be included in this presentation? (Select all that apply.) a. Drink one cup of caffeinated coffee each day b. Ensure an adequate intake of calcium each day c. Participate in weight-bearing exercise every day d. Wear well-supporting nonskid shoes at all times e. Consider participating in resistance exercise training Common Endocrine/Metabolic Complaints Multiple Choice Identify the choice that best completes the statement or answers the question. 1. A patient is 66 inches in height, weighing 200 pounds, and newly diagnosed with type 2 diabetes mellitus (DM). Her fasting plasma glucose level is 215 mg/dL. What is the best initial treatment? a. No treatment at this time b. Diet and exercise for 6-week trial c. Diet, exercise, and oral medication d. Diet, exercise, and exogenous insulin 2. The clinician suspects that a client seen in the office has hyperthyroidism. Which of the following tests should the clinician order on the initial visit? a. High sensitivity thyroid-stimulating hormone (TSH) and free T4 b. Free T4 and serum calcium c. Free T3 and T4 d. TSH and thyroxin antibodies 3. A patient with type 2 diabetes asks the clinician why she needs to exercise. In order to answer her, the clinician must understand that exercise has what effect on the patient with type 2 diabetes? a. Reduces postprandial blood glucose b. Reduces triglycerides and increases high-density lipoprotein (HDL) c. Reduces total cholesterol d. All of the above 4. A patient with type 1 diabetes comes to the clinic complaining of feeling nervous and clammy. He states that he took his insulin this morning but was late for work and did not eat breakfast. Which action should the clinician take first? a. Check his blood sugar. b. Have him drink 4 ounces of juice. c. Call 911. d. Ask him about his usual eating habits. 5. A patient with type 2 diabetes comes to the clinic after reading about metformin in a magazine. Which of the following conditions that the patient also has would be a contraindication to taking metformin? a. Ketoacidosis b. Cirrhosis c. Hypoglycemic episodes d. All of the above 6. A 25-year-old patient presents to the clinic with fatigue, cold intolerance, weight gain, and constipation for the past 3 months. On physical examination, the clinician notices a sinus bradycardia; muscular stiffness; coarse, dry hair; and a delay in relaxation in deep tendon reflexes. Which of the following tests should be ordered next? a. Serum calcium b. TSH c. Electrolytes d. Urine specific gravity 7. The clinician has been doing diabetic teaching for a patient with type 1 diabetes. Which of the following statements by the patient would indicate that teaching has been effective? a. “As long as I don’t need glasses, I don’t have to worry about going blind.” b. “I know I need to have my eyes checked every year.” c. “My optometrist checks my eyes.” d. “I will see my eye doctor when my vision gets blurry.” 8. A 64-year-old man with type 2 diabetes presents to the clinic with the complaint of “my feet feel like they are on fire.” He has a loss of vibratory sense, +1 Achilles reflex, and a tack embedded in his left heel. Which of the following would be an appropriate treatment? a. Tricyclic antidepressants b. Capsaicin cream c. Vitamin B12 injections d. Insulin 9. After removing a tack from a type 2 diabetic’s heel and evaluating the site for infection, what is the best plan for this patient? a. Suggest she use a heating pad to improve circulation. b. Refer to a podiatrist for a foot care treatment plan. c. Send her for acupuncture treatments. d. All of the above 10. Joyce is seen in the clinic complaining of vague symptoms of nervousness and irritability. She says that her hair will not hold a permanent wave anymore. On physical examination, the clinician finds an irregular heartbeat and brisk reflexes. The differential diagnosis should include which of the following conditions? a. Myxedema b. Thyrotoxicosis c. Cushing’s syndrome d. Pan-hypopituitarism 11. The patient is prescribed radioactive iodine (RAI) and asks the clinician how this drug works. The clinician’s response should include which of the following data? a. RAI prevents the peripheral conversion of T4 to T3. b. RAI binds free T4. c. RAI destroys thyroid tissue. d. RAI reduces freely circulating iodine. 12. A patient is diagnosed with hypothyroidism. Which of the following electrocardiogram changes should the clinician expect as a manifestation of the disease? a. Sinus bradycardia b. Atrial fibrillation c. Supraventricular tachycardia d. U waves 13. After 6 months of Synthroid therapy, the clinician should expect which of the following in the repeat thyroid studies? a. Elevated TSH b. Normal TSH c. Low TSH d. Undetectable TSH 14. Which of the following laboratory findings should the clinician expect in a patient with untreated Graves’ disease? a. Elevated TSH b. Elevated T4 c. Elevated thyrotropin-releasing hormone (TRH) d. All of the above 15. The clinician prescribes glipizide (Glucotrol) for a diabetic patient. Which statement made by the patient would indicate that your teaching has been effective? a. “I’ll take my pill at least 30 minutes before breakfast.” b. “I’ll take my Glucotrol before bedtime.” c. “It is important to take my medication right after I eat.” d. “Since I only like to eat two meals a day, I can take the pill between my meals.” 16. A diabetic patient asks the clinician why he needs to check his blood sugar at home even when he feels good. Which of the following responses would be most appropriate? a. “Control of glucose will help postpone or delay complications.” b. “Regularly checking blood sugar will help establish a routine.” c. “Monitoring glucose will promote a sense of control.” d. All of the above 17. How often should the clinician examine the feet of a person with diabetes? a. Once a year b. Every 6 months c. Every 3 months d. Every visit 18. The clinician sees a patient who is 5 feet tall and weighs 150 pounds. How would the clinician classify this patient? a. Overweight b. Mild obesity c. Moderate obesity d. Morbid obesity 19. Mr. S presents in the clinic with pain, tenderness, erythema, and swelling of his left great toe. The clinician suspects acute gout. Which of the following should the clinician expect in the initial test results for this patient? a. Elevated uric acid level b. Elevated blood urea nitrogen c. Decreased urine pH d. Decreased C-reactive protein 20. Mr. W, 53 years old, is seen in the clinic with concerns about his left foot. He has a 40-year history of type 1 diabetes with “fairly good” control on twice-daily insulin. He denies injury but states that he tripped a few months ago and that his foot is sore when he walks. Physical examination reveals an edematous, erythremic, and warm foot. There is a superficial ulcer on the plantar surface. Which of the following is the most likely diagnosis? a. Fallen arch b. Arthritis c. Charcot joint d. Sprained ankle 21. Which of the following tests should you order to confirm Mr. W’s diagnosis? a. Bone scan b. Computed tomography (CT) scan c. X-ray of the foot d. Culture of the ulcer 22. A vegetarian patient with gout asks the clinician about food he should avoid. The clinician should advise the patient to avoid which of the following foods? a. Rice b. Carrots c. Spinach d. Potatoes 23. The clinician should question the patient with suspected gout about use of which of these medications? a. Low-dose aspirin b. Thiazide diuretics c. Ethambutol d. All of the above 24. The clinician finds numerous nodules on the thyroid of a 65-year-old woman. The clinician suspects thyroid cancer. Which of the following data would be most significant for this patient? a. A history of tonsillectomy in the 1940s b. Recent exposure to mumps c. Vegetarian diet d. Allergy to iodine 25. Which of the following is essential for diagnosing thyroid cancer? a. Fine needle aspiration b. Thyroid ultrasound c. CT scan d. Magnetic resonance imaging 26. Which of the following are common signs of type 2 DM? a. Anorexia b. Recurrent yeast infection c. Weight gain d. Elevated HDL cholesterol 27. Which of the following medications can cause hyperglycemia? a. Prednisone b. Metformin c. Synthroid d. Cephalexin 28. Which of the following is diagnostic for diabetes mellitus? a. A1C 7.0 on one occasion b. Fasting blood sugar (FBS) of 100 mg/dL on two occasions c. Random glucose of 200 mg/dL on two occasions d. Two-hour post-load plasma glucose of 300 mg/dL on one occasion 29. Which of the following medications for type 2 diabetes mellitus should not be prescribed during pregnancy? a. Insulin b. Metformin c. Glucotrol d. Precose 30. A 35-year-old woman presents with symptoms of hypoglycemia. There is no history of diabetes mellitus. Which of the following should be included in the differential diagnosis? a. Anxiety disorder b. Pheochromocytoma c. Psychosis d. All of the above True/False Indicate whether the statement is true or false. 1. Metformin is the first line of pharmacologic treatment for type 2 DM. 2. Fruit juice with added sugar is the treatment of choice for anyone experiencing hypoglycemia. 3. Lifestyle modification is the treatment of choice for metabolic syndrome. 4. A BMI of 29 kg/m2 is considered obesity. 5. Urine-free cortisol is one of four diagnostic tests recommended for Cushing’s syndrome. [Show More]

Last updated: 1 year ago

Preview 1 out of 530 pages

Add to cart

Instant download

document-preview

Buy this document to get the full access instantly

Instant Download Access after purchase

Add to cart

Instant download

Reviews( 0 )

$15.00

Add to cart

Instant download

Can't find what you want? Try our AI powered Search

OR

REQUEST DOCUMENT
53
0

Document information


Connected school, study & course


About the document


Uploaded On

Mar 28, 2021

Number of pages

530

Written in

Seller


seller-icon
CHIEFCREATOR

Member since 3 years

23 Documents Sold


Additional information

This document has been written for:

Uploaded

Mar 28, 2021

Downloads

 0

Views

 53

Document Keyword Tags

Recommended For You

Get more on TEST BANK »

$15.00
What is Browsegrades

In Browsegrades, a student can earn by offering help to other student. Students can help other students with materials by upploading their notes and earn money.

We are here to help

We're available through e-mail, Twitter, Facebook, and live chat.
 FAQ
 Questions? Leave a message!

Follow us on
 Twitter

Copyright © Browsegrades · High quality services·